Location via proxy:   [ UP ]  
[Report a bug]   [Manage cookies]                
Download as pdf or txt
Download as pdf or txt
You are on page 1of 32

Solutions to exercises and problems in Lee’s Introduction to

Smooth Manifolds
Samuel P. Fisher
August 22, 2020

1 Topological Manifolds
Exercise 1.1. Show that equivalent definitions of manifolds are obtained if instead of allowing U to be
homeomorphic to any open subset of Rn , we require it to be homeomorphic to an open ball in Rn , or to Rn
itself.
Every open ball is homeomorphic to the the open unit ball centred at the origin (we can map any two
balls onto each other by linear scaling and a translation, both of which are continuous with continuous
inverses). Note that open unit ball is homeomorphic to Rn itself via the homeomorphism
x
f : int(Bn ) → Rn , x 7→ .
1 − kxk
Hence, it is enough to show that we obtain an equivalent definition of a topological manifold if we require
that U be homeomorphic to an open ball. First, suppose that at every p ∈ M , there is an open neighbourhood
U such that U b ∈ Rn is open and φ : U → Ub is open. Then φ(p) ∈ B ⊆ U b , where B is some open ball. Then
−1
φ (B) is an open subset of M containing p that is homeomorphic to an open ball in Rn . Conversely, if
every point p ∈ M is contained in a neighbourhood homeomorphic to an open ball in Rn , then M already
satisfies the given definition.
Exercise 1.6. Show that RPn is Hausdorff and second-countable, and is therefore a topological n-manifold.
Let [x] and [y] be distinct points of RPn , that is, distinct 1-dimensional subspaces of Rn+1 , and let
them be spanned by the unit vectors x and y respectively. Since Sn is Hausdorff, it is not hard to see that
we can find pairwise disjoint open sets U, U , V, V ⊆ Sn such that x ∈ U, −x ∈ U e , y ∈ V, −y ∈ Ve , where
A = −a : a ∈ A. Let Ub =U ∪U e and Vb = V ∪ Ve . Define
p
φ : Rn \ {0} → Sn , p 7→ .
kpk

We claim that U = π(φ−1 (U b )) and V = π(φ−1 (Vb )) are disjoint and contain [x] and [y] respectively, and that
U and V are open. It is clear that [x] ∈ U and [y] ∈ V . Let [p] ∈ U ∩ V . Then [p] = π(u) = π(v) for some
u ∈ φ−1 (U b ), v ∈ φ−1 (Vb ). Then u = λv for some λ ∈ R \ {0}. Hence, φ(u) = ±φ(v). But this implies that
φ(u) ∈ U b ∩ Vb , a contradiction. We conclude that RPn is Hausdorff.
Let B be a countable basis for Rn . We claim that π(B) = {π(B) : B ∈ B} is a basis for RPn . Define
ft : Rn \ {0} → Rn \ {0}, p 7→ tp for every t ∈ R \ {0}. Note that ft is continuous and S has a continuous inverse
f1/t . Hence, for if U is open, then ft (U ) is also open. We claim that π −1 (π(U )) = t∈R\{0} ft (U ) for every
open set U ∈ Rn \ {0}. Let p ∈ π −1 (π(U )). Then π(p) ∈ π(U ), implying that there is a uS∈ U such that
u spans the same vector space as p. Hence, p = λu for some non-zero λ, and therefore p ∈ t∈R\{0} ft (U ).
S
Conversely, suppose that p ∈ t∈R\{0} ft (U ). Then p = fλ (u) = λu for some u ∈ U and some non-zero
λ. Hence, π(p) = π(λu) = π(u), so p ∈ π −1 (π(U )), proving the claim. We are now ready to prove second-
countability, equivalently to prove that π(B) is a basis. Let [p] ∈ π(B1 )∩π(B2 ) for two basis sets B1 , B2 ∈ B.
Then p ∈ π −1 (π(B1 )) ∩ π −1 (π(B2 )), which is open by our previous work. Since this set is nonempty, there
is a basis set B3 contained in π −1 (π(B1 )) ∩ π −1 (π(B2 )). Then π(B3 ) ⊆ π(B1 ) ∩ π(B2 ), showing that π(B)
is a basis, and therefore showing that RPn is a manifold.

1
Exercise 1.7. Show that RPn is compact. [Hint: show that the restriction of π to Sn is surjective.]
Let [p] = π(p) ∈ RPn . Then π(p/kpk) = π(p), and p/kpk ∈ Sn . Then the restriction of π to Sn is
surjective. Since Sn is compact, and the image of a compact set under a continuous function is compact,
RPn is compact.

Exercise 1.14. Suppose X is a locally finite collection of subsets of a topological space M .


(a) The collection {X : X ∈ X} is also locally finite.
S S
(b) X∈X X = X∈X X.
S S
Let’s prove (b) first. Let x ∈ X∈X X. Then every open set U containing x intersects X∈X X. By local
finiteness, choose an open set U containing x intersecting X1 , . . . , Xn ∈ X. Suppose x ∈
S
/ X∈X X. Then, for
each i ∈ {1, . . . , n}, there is an open set Ui that
S contains x but does not intersect Xi . Then U ∩ U1 ∩ · · · ∩ Un
contains x, is open, but does not intersect X∈X X, which is a contradiction. Conversely, suppose that
x ∈ X∈X X. Then x ∈ X for some X ∈ X; thus, every open set containing x intersects X, and therefore
S
S S
intersects X∈X X as well. Hence, x ∈ X∈X X.
For (a), let x ∈ M , and let U be an open set containing x and intersecting X1 , . . . , Xn ∈ X. Let
X
e = X \ {X1 , . . . , Xn }. Since U contains x but does not intersect any of the sets in X,
e we have x ∈ S
/ X∈X e X.
S S
Hence, U \ X∈X e X = U \ X∈Xe X is an open set containing x and intersecting only sets in {X1 , . . . , X n }.
Therefore, {X : X ∈ X} is locally finite.
Exercise 1.18. Let M be a topological manifold. Two smooth atlases for M determine the same smooth
structure if and only if their union is a smooth atlas.
(⇒) Let A1 and A2 be two atlases for M determining the same smooth structure. By Proposition 1.17
(a), these two atlases are contained in a unique maximal smooth atlas A. Since any two charts in A are
smoothly compatible, then any two charts in A1 ∪ A2 are smoothly compatible. Since each Ai covers M ,
A1 ∪ A2 covers M and is therefore a smooth atlas.
(⇐) Let A1 and A2 be smooth atlases and let A1 ∪A2 be a smooth atlas. By part (a), A1 and A2 are each
contained in a maximal smooth atlas. Since A1 and A2 are each contained in A1 ∪ A2 , they are contained
in the same maximal atlas, and therefore they determine the same smooth structure on M .

Exercise 1.20. Every smooth manifold has a countable basis of regular coordinate balls.
We’re not going to worry about our coordinate balls being centred at 0, since a ball in Rn can always be
mapped to a ball centred at 0 in Rn via a translation. Let M be a topological n-manifold, and let {Ui , φi }
be an atlas of charts for M . We may assume that the atlas is countable, since M is second-countable. Note
that φi (Ui ) ⊆ Rn has a countable basis of regular coordinate balls, namely those of the for Br (x), such that
x ∈ Qn , r ∈ Q, and such that Br (x) ⊆ Br0 (x) ⊆ φi (Ui ) for some r0 > r. We claim that B = {φ−1 i (Br (x))}
forms a basis of regular coordinate balls for the topology on M . Let B1 , B2 ∈ B intersect non-trivially.
Then B1 ∩ B2 ⊆ Ui for some i, and φi (B1 ∩ B2 ) ⊆ φi (Ui ) ⊆ Rn . There is a rational coordinate ball B
whose closure is contained in another coordinate ball with the same centre. Hence, φ−1 i (B) = B1 ∩ B2 .
Hence, B is a basis. It remains to show that the elements of B are regular. Let φ−1 i (Br (x)) ∈ B. Then
φi (φ−1 −1 −1 −1
i (Br (x))) = Br (x) ⊆ Br 0 (x) ⊆ φi (Ui ). Hence, φi (Br (x)) ⊆ φi (Br 0 (x), so φi (Br (x)) is a regular
coordinate ball.
Exercise 1.39. Let M be a topological n-manifold with boundary.

(a) IntM is an open subset of M and a topological n-manifold without boundary.


(b) ∂M is a closed subset of M and a topological (n − 1)-manifold without boundary.
(c) M is a topological manifold if and only if ∂M = ∅.
(d) If n = 0, then ∂M = ∅ and M is a 0-manifold.

2
For (a), let x ∈ IntM . Then x ∈ U , where (U, φ) is an interior chart. We claim that U ⊆ IntM . Let
y ∈ U . Then φ(y) ∈ φ(U ) ⊆ Rn . Since φ(U ) is open, there is an open set Vb such that φ(y) ∈ Vb ⊆ φ(U ).
Then y ∈ φ−1 (Vb ) ⊆ U . Therefore, (φ−1 (Vb ), φ|φ−1 (Vb ) ) is an interior chart containing y, so y ∈ IntM . Hence,
U ⊆ IntM , which proves that IntM is open. Since IntM is a subspace of M , which is Hausdorff and second-
countable, we have that IntM is Hausdorff and second-countable. By definition, every point of IntM is
contained in an interior chart, so IntM is a topological n-manifold.
For (b), by the theorem on topological invariance of the boundary, ∂M = M \ IntM . By part (a),
∂M is closed. ∂M is Hausdorff and second-countable since it is a subspace of M , which is Hausdorff and
second-countable. It remains to show that every x ∈ ∂M is in some chart (U, φ) with φ(U ) ⊆ Rn−1 . Let
x ∈ ∂M , and let (V, ψ) be a boundary chart containing x. Then ψ(x) ∈ ∂Hn . Let U b = ψ(V ) ∩ ∂Hn . Note
that Ub contains x, and is open in ∂H ∼n
=R n−1
. Let U = ψ (U −1 b ) = V ∩ ∂M . Then U is open in ∂M , and
(U, ψ|U ) is a chart for x ∈ ∂M . This proves that ∂M is an (n − 1)-manifold.
For (c), let M be a topological manifold. By definition, every point is in an interior chart. By the theorem
on topological invariance of the boundary, ∂M = ∅. Conversely, let ∂M = ∅. Again, by the theorem on the
topological invariance of the boundary, every point is then an interior point. M then satisfies the definition
of a topological manifold.
For (d), let M be a 0-manifold. Then every point p ∈ M is in a chart ({p}, φ), where φ : {p} → R0 is a
homeomorphism. Hence, every point of M is an interior point, so ∂M = ∅. By part (c), M is a 0-manifold.
Exercise 1.41. Let M be a topological manifold with boundary.
(a) M has a countable basis of precompact coordinate balls and half-balls.
(b) M is locally compact.
(c) M is paracompact.
(d) M is locally path-connected.
(e) M has countably many components, each of which is an open subset of M and a connected topological
manifold with boundary.
(f) The fundamental group of M is countable.
For (a), let {(Ui , φi )} be a collection of charts covering M . We may assume that this collection is
countable, since M has a countable basis by definition. Say (U, φ) is an interior chart. Then φ(U ) ⊆ Rn has
a countable basis of balls whose closures lie in φ(U ). Similarly, if (U, φ) is a boundary chart, then φ(U ) has
a countable basis of balls and half-balls whose closures all lie in φ(U ). Let B be the collection of preimages
of all the coordinate balls and half-balls discussed above, under the respective coordinate maps. Then B is
a countable basis for M . It remains to show that the sets in B are precompact. Let B ∈ B, and let φ be
its coordinate map. Then B ⊆ U , where (U, φ) is a coordinate chart (the φ in this chart is an extension of
the φ in (B, φ)). Then φ(B) = φ(B) ⊆ φ(U ) is compact, so B is compact, where B is the closure of B in U .
Since M is Hausdorff, B is closed in M . It follows that B is also the closure of B in M . This proves (a).
For (b), part (a) implies that every x ∈ M is contained in a precompact coordinate ball. Hence, M is
locally compact.
For (c), let X be an open cover of M , and let B be an arbitrary basis for the topology on M . We will
prove the stronger result that X has a countable, open refinement consisting of elements of B. Let {Kj }∞ j=1
S∞
be an exhaustion of M by compact sets; that is X = j=1 Kj and Kj ⊆ IntKj+1 . Let Aj = Kj+1 \ IntKj
and let Bj = IntKj+2 \ Kj−1 . Then Aj is compact and Bj is open, and Aj ⊆ Bj . Every x ∈ Aj is contained
in some basis set B ⊆ X ∩ Bj . Since these basis sets cover the compact set Aj , there is a finite cover of Aj .
Taking the union of these basis sets over j ∈ N, we obtain a countable refinement of X consisting of sets in
B. The cover is locally finite, since the basis sets are all contained in some Bj , Bj and Bj 0 don’t intersect if
|j − j 0 | > 2.
For (d), by part (a) M has a basis of coordinate balls and half-balls, which are all path-connected. Hence,
M is locally path-connected.
For (e), let {Ci } be the collection of connected components of M . Then {Ci } is an open cover of M . Since
M is second-countable, {Ci } has a countable subcover. But since the Ci are disjoint, we must have that {Ci }
is countable to begin with. Hence, M has countably many components. By definition, each component is an

3
open subset. As subspaces of M , the Ci are Hausdorff and second-countable. Let x ∈ Ci and let (U, φ) be a
chart containing x. Then (U ∩ Ci , φ|U ∩Ci ) is a chart in Ci containing x. Thus, Ci is a connected topological
manifold with (possibly empty) boundary.
For (f), let B be a countable basis of coordinate balls and half-balls. For any B1 , B2 ∈ B, the intersection
B1 ∩ B2 has countably many components. To see this, note that B1 ∩ B2 is open in M , and therefore its
components are open in M . Moreover, every component is itself a connected manifold with boundary, and
therefore is locally path-connected. Since connected and locally path-connected spaces are path-connected,
every component is path-connected. Since each component contains a distinct basis set, B1 ∩B2 has countably
many components. Let X be a set containing a single point from each component of B1 ∩ B2 for every pair
B1 , B2 ∈ B. For every pair x, x0 ∈ X contained in some B ∈ B, let hB x,x0 be a fixed path from x to x in
0

B. Select a point p ∈ X as the basepoint of the fundamental group π1 (M, p). Define a special loop to be a
loop starting at p that is the concatenation of paths of the form hB x,x0 . Since the set of all special loops is
countable, it suffices to show that every loop based at p is homotopic to some special loop. Let f : [0, 1] → M
be an arbitrary loop based at p. Since f ([0, 1]) is compact, there are finitely many basis sets B1 , . . . , Bk ∈ B
that cover it. Hence, there exist 0 = a0 < a1 < · · · < ak = 1 such that f ([ai−1 , ai ]) ⊆ Bi for each i. Each
f (ai ) is contained in the same component of Bi ∩ Bi+1 as some xi ∈ X. Let gi be a path from xi to f (ai ),
where g0 and gk are both equal to the constant path at p. Let fi be the path obtained by restricting f to
[ai−1 , ai ],

f ' f1 · f2 · · · · · fk ' g0 · f1 · ·g1 · g1 · f2 · g2 · · · · · gk−1 · fk · gk .

Since each Bi is simply connected, and the endpoints of gi−1 · fi · gi are xi−1 and xi , gi−1 · fi · gi is homotopic
to hB
xi−1 ,xi . Hence, f is homotopic to a special loop, concluding the proof.
i

Exercise 1.42. Show that every smooth manifold with boundary has a countable basis consisting of regular
coordinate balls and half-balls
This is a straightforward adaptation of Exercise 1.20.
Exercise 1.43. Show that the smooth manifold chart lemma (Lemma 1.35) holds with “ Rn ” replaced with
“ Rn or Hn ” and “smooth manifold” replaced by “smooth manifold with boundary.”
Just make the same replacements in the proof of Lemma 1.35. The only external result that is used is
Exercise A.22, which is a result for general topological spaces.
Exercise 1.44. Suppose M is a smooth n-manifold with boundary, and U is an open subset of M . Prove
the following statements:
(a) U is a topological n-manifold with boundary, and the atlas consisting of all smooth charts (V, φ) for
M such that V ⊆ U defines a smooth structure on U . With this topology and smooth structure, U is
called an open submanifold with boundary.
(b) If U ⊆ IntM , then U is actually a smooth manifold (without boundary); in this case, we call it an
open submanifold of M .
(c) IntM is an open submanifold of M (without boundary).
For (a), every point p ∈ U is contained in the domain of some chart (V, φ). Then (U ∩ V, φ|U ∩V ) is a
chart whose domain is contains p and is contained in U . Thus, U is a topological n-manifold with boundary.
These charts clearly cover U , so the form an atlas. They are smoothly compatible, since this atlas is a subset
of the maximal smooth atlas for M . Hence, this collection of charts defines a smooth structure on U .
For (b), we need to show that all of the charts are in fact interior charts. Let (V, φ) be a smooth chart
with V ⊆ U . If φ(V ) ⊆ Hn with some p ∈ V such that φ(p) ∈ ∂Hn . Then p is a boundary point and an
interior point, which is impossible by the theorem on the topological invariance of the boundary.
For (c), by exercise 1.39 IntM is open in M . Take U = IntM in part (b).
Problem 1.1. Let X be the set of all points (x, y) ∈ R2 such that y = ±1, and let M be the quotient of X
by the equivalence relation generated by (x, −1) ∼ (x, 1) for all x 6= 0. Show that M is locally Euclidean and
second-countable, but not Hausdorff. (This space is called the line with two origins.)

4
Let π : X → M, x 7→ [x]. Let [(x, 1)] ∈ M for some x (we are including the possibility that x = 0, the
[(0, −1)] case is similar). Then there is an open set (a, b) × {1} in X such that x ∈ (a, b). We claim that the
map φ : π((a, b) × {1}) → (a, b), [(y, ±1)] 7→ y is a homeomorphism. If [(y, ±1)] = [(y 0 , ±1)], then y = y 0 , so φ
is well-defined. Let φ([(y, ±1)]) = φ([(y 0 , ±1)]) ⇒ y = y 0 . If y = y 0 = 0, then [(y, ±1)] = [(y 0 , ±1)] = [(0, 1)],
since the points [(y, ±1)] and [(y 0 , ±1)] must be in π((a, b) × {1}). Otherwise, we can ignore the ±1 since it
does not affect the equivalence class, so [(y, ±1)] = [(y 0 , ±1)]. Thus, φ is injective. It is also clearly surjective,
and thus bijective. Let U ⊆ (a, b) be open. Then φ−1 (U ) = π(U × {1}). Since π −1 (π(U × {1})) = U × {±1}
is open in X, we have that π(U × {1}) is open in M . Let U ⊆ π((a, b) × {1}) be open in M . Then U × {±1}
is open in X, and therefore U is open in (a, b) ⊆ R. Thus, π is a homeomorphism and M is locally Euclidean.
Let U ∈ X be open. Then π −1 (π(U )) = U ∪ r(U ), where r is the reflection across the x-axis. Since
U ∪ r(U ) is open, π −1 (π(U )) is open. Hence, π is an open map. By the same argument given in Exercise
1.6, π(B) is a basis for M if B is a basis for X. Since X is a subspace of a second-countable space, M is
second-countable.
Let U and V be open sets in M containing [(0, 1)] and [(0, −1)] respectively. Hence, π −1 (U ) and π −1 (V )
are open sets in X containing (0, 1) and (0, −1) respectively. Hence, U and V contain subsets (−, ) × {1}
and (−, ) × {−1}, respectively, for some  > 0. Then π((−, ) × {1}) ⊆ U and π((−, ) × {−1}) ⊆ V .
However, these two sets have non-trivial intersection. We conclude that [(0, 1)] and [(0, −1)] cannot be
separated by open sets, and therefore M is not Hausdorff.
Problem 1.2. Show that a disjoint union of uncountably many copies of R is locally Euclidean and Haus-
dorff, but not second-countable.
Let M be a disjoint union of uncountably many copies of R. Let x ∈ M . Let R be the component
containing x. Then the homeomorphism φ : R → R defines a chart whose domain contains x. Therefore, M
is locally Euclidean. The disjoint union of Hausdorff spaces is Hausdorff, so M is Hasudorff. Since manifolds
have countably many components, M cannot be a manifold. We conclude that M is not second-countable
since it satisfies the other conditions of the definition.
Problem 1.3. A topological space is said to be σ-compact if it can be expressed as a union of countably
many compact spaces. Show that a locally Euclidean Hausdorff space is a topological manifold if and only if
it is σ-compact.
(⇒) Let M be a topological manifold. Then M is second countable. By Lemma 1.10, M has a countable
basis of precompact coordinate balls. Then M is the union of the closures of these basis sets, which are
compact.
(⇐) Let M be locally Euclidean, Hausdorff, and σ-compact. Let {Ki }i∈N be a countable cover of M
by compact sets. Since M is locally Euclidean, M has an open cover of coordinate balls. Each Ki is then
covered by finitely many such coordinate balls, so we can assume that M is covered by countably many
S ⊆−1
coordinate balls. Let (B, φB ) be such a coordinate ball. Then φ(B) Rn has a countable basis BB . Let
−1
φB (BB ) be collection of preimages of sets in BB under φB . Then B φB (BB ) is a basis for M . Therefore,
M is second-countable.

Problem 1.4. Let M be a topological manifold, and let U be an open cover of M .


(a) Assuming that each set in U intersects only finitely many others, show that U is locally finite.
(b) Give an example to show that the converse to (a) may be false.
(c) Now that the sets in U are precompact in M , and prove the converse: if U is locally finite, then each
set in U intersects only finitely many others.

For (a), let p ∈ M be arbitrary. Then p ∈ U for some U ∈ U, and U is a neighbourhood of p that
intersects at most finitely many of the other sets in U.
For (b), let M = R, and let U contain R, and the intervals (i, i + 1) for each i ∈ Z. Then U is locally finite
since every x ∈ R is contained in some open interval of unit length, and every such open interval intersects
at most three of the sets in U.
For (c), let U ∈ U. For every x ∈ U , let Ux be an open set containing x such that Ux intersects finitely
many sets in U. Then {Ux }x∈X is an open cover for U , which is compact. Hence, it has a finite subcover

5
(1) (k) (i)
{Ux , . . . , Ux }. Since each Ux intersects finitely many sets in U and they cover U , we conclude that U
intersects finitely many sets in U.
Problem 1.5. Suppose M is a locally Euclidean Hausdorff space. Show that M is second-countable if and
only if it is paracompact and has countably many connected components.
(⇒) If M is second-countable, it is a topological manifold. Hence, it is countably many components and
is paracompact by the results from the section.
(⇐) Let M be paracompact and have countably many components. We can assume that M is connected:
if each connected component has a countable basis, and there are countably many components, then M has
a countable basis.
Let U be an open cover of M by coordinate charts and let (U, φ) be such a chart. Note that φ(U ) has a
basis of precompact balls, where the closures of said balls lie in φ(U ). Pulling back these balls to M yields
a covering of M by balls whose closure in U is compact, and thus closed because M is Hausdorff. Thus,
the closure of a coordinate ball in U is the same as its closure in M , and therefore we have an open cover
of precompact coordinate charts for M . Since M is paracompact, the open cover constructed has a locally
finite open refinement. Since M is Hausdorff, the this locally finite refinement also consists of precompact
coordinate charts. Call this open cover U.
Let U ∈ U be arbitrary, and define V0 = {U }. By part (c) of the preceding problem, U intersects
finitely many sets in U. Denote the collection of these sets W1 , and define V1 = W1 \ V0 . Suppose we have
define Vn ⊆ U. Let Wn+1 be the collection of all sets in U that intersect sets in V0 ∪ · · · ∪ Vn , and define
V
Sn+1 = Wn+1 \ (V0 ∪ · · · ∪ Vn ). By construction, each Vi contains finitely many sets. We will show that

i=0 Vi covers M . Since M is locally Euclidean, it is locally path-connected, and because we are assuming
that M is connected, it is path-connected. Let y ∈ M and x ∈ U be arbitrary, and let f : [0, 1] → M be a
path from x to y. Since f ([0, 1]) is compact, it is covered by finitely many of the sets in U, and therefore
there are numbers 0 = a0 < a1 < · · · < ak = 1 such that f ([ai−1 , ai ]) ⊆ Ui for some Ui ∈ U. Then
f ([a0 , a1 ]) = f ([0, a1 ]) ⊆ U1 = U . Suppose that we have shown that f ([aj−1 , aj ]) is contained in the union
of the sets in V0 ∪ · · · ∪ Vj−1 . Then f (aj ) ∈ V for some V ∈ V0 ∪ · · · ∪ Vj−1 . Hence, f ([aj , aj+1 ]) must be
contained in the union of sets in V0 ∪ · · · ∪ Vj , since it intersects the sets in V0 ∪ · · · ∪ Vj−1 . Therefore, y is
contained in a set in some Vk , which proves the claim. Hence, we have a countable cover of M by precompact
coordinate charts. Each of these charts has a countable basis, and the union of all these bases is a basis for
the topology on M . Hence, M is second countable.
Problem 1.6. Let M be a nonempty topological manifold of dimension n ≥ 1. If M has a smooth structure,
show that it has uncountably many distinct ones.
s−1
Following the hint, for every s > 0 we show that Fs : Bn → Bn , x 7→ |x| x is a homeomorphism, and a
s
diffeomorphism if and only if s = 1. Note that Fs is not defined at x = 0 for s ≤ 1. However, |Fs (x)| = |x| ,
s
so defining Fs (0) = 0 makes Fs a continuous function. Moreover, |Fs (x)| = |x| ≤ 1, so the image of Fs does
1
−1
indeed lie in Bn . The inverse of Fs is given by Fs−1 (x) = |x| s x, where we define Fs−1 (0) = 0. Clearly, Fs−1
is continuous, so Fs is a homeomorphism for each s > 0.
Away from x = 0, Fs is a diffeomorphism since Fs and Fs−1 are products or quotients of smooth non-
vanishing functions. Moreover, F1 = id is a diffeomorphism. Now suppose that s 6= 1. It suffices to show
that Fs is not smooth for s < 1, since the same proof will show that Fs−1 is not smooth for s > 1. If Fs is
smooth, then it should have continuous derivatives of all orders. However, the partial derivative of the first
component of Fs with respect to the first variable is
∂ s−1 s−3 s−1
|x| x1 = (s − 1)|x| x21 + |x| .
∂x1
To see that this quantity is not continuous at 0, we can take its limit as x → 0 along the x1 -axis. Setting
x2 = · · · = xn = 0, we find
∂ s−1 s−3 s−1 s−1
|x| x1 = (s − 1)|x1 | x21 + |x1 | = s|x1 | → ∞,
∂x1
since s < 1. This concludes our proof of the statement in the hint.

6
Given that M has a smooth structure, let A be a locally finite atlas for M . Let x ∈ M and let x be
contained in the domains of the charts (U1 , φ1 ), . . . , (Uk , φk ). Define Let A0 = A, and inductively define
(
Ai−1 \ {(Ui , φi )} if Ui−1 is covered by the domains of other charts in Ai−1
Ai =
Ai−1 otherwise

for i = 1, . . . , k. By induction, Ak is an atlas for M . Hence, the charts in Ak cover M . Since x ∈ M , there
must be at least one i ∈ {1, . . . , k} such that (Ui , φi ) ∈ Ak . Hence, Ui is not covered by the domains in of
charts in Ai−1 , and therefore is not covered by the domains of charts in Ak ⊆ Ai−1 . Thus, there is a p ∈ Ui
that is covered only by Ui . Moreover, φi (Ui ) \ {φi (p)} can be covered by open balls Bα = Brα (xα ) entirely
contained in φi (Ui ) \ {φi (p)}. There is also a ball B = Br (φi (p)) ⊆ φi (Ui ). Pulling back these balls, we get
that A0 = Ak \ {(Ui , φi )}] ∪ {(φ−1 −1
i (Bi ), φi )} ∪ {(φi (B), φi )} is a smooth atlas for M such that p is only
−1
contained the domain of (φi (B), φi ). The coordinate maps are smoothly compatible, since the new maps
are obtained by restricting φ, which is smoothly compatible with all of the other coordinate maps. There
is a diffeomorphism f : B → Bn obtained by translating and then scaling. Replacing (φ−1 i (B), φi ) with
(φ−1
i (B), f ◦ φ i ) in A 0
, we obtain a new smooth atlas Z.
For every s > 0, define Zs to be the set of charts in Z, except (φ−1 −1
i (B), f ◦φi ) is replaced with (φi (B), Fs ◦
f ◦ φi ). We claim that each Zs is a smooth atlas, and that Zs is not smoothly compatible with Zs0 for s 6= s0 .
Clearly, every pair of charts not including (φ−1 i (B), Fs ◦ f ◦ φi ) is smoothly compatible. Now consider the
pair of charts (φ−1 i (B), Fs ◦ f ◦ φ i ) and (U, φ) and suppose that φ−1 i (B) ∩ U 6= ∅. The transition function
−1 −1
is then Fs ◦ f ◦ φi ◦ φ . Since φi (B) ∩ U does not contain p and Fs is a diffeomorphism away from 0, we
have that the transition function is a diffeomorphism. This shows that Zs is a smooth atlas, and therefore
determines a smooth structure for each s. For s 6= s0 , the charts (φ−1 −1
i (B), Fs ◦f ◦φi ) and (φi (B), Fs0 ◦f ◦φi )
are not smoothly compatible, since

Fs ◦ f ◦ φi ◦ (Fs0 ◦ f ◦ φi )−1 = Fs ◦ f ◦ φi ◦ φ−1


i ◦f
−1
◦ Fs−1
0 = Fs ◦ Fs−1
0 = Fs/s0

(the last equality is a standard computation) is not a diffeomorphism. We conclude that Zs and Zs0 are not
smoothly compatible and therefore there are uncountably many different smooth structures on M .
Problem 1.7. Let N denote the north pole (0, . . . , 0, 1) ∈ Sn ⊆ Rn+1 , and let S denote the south pole
(0, . . . , 0, −1). Define the stereographic projection σ : Sn \ {N } → Rn by

(x1 , . . . , xn )
σ(x1 , . . . , xn+1 ) = .
1 − xn+1
Let σ̃(x) = −σ(−x) for x ∈ Sn \ {S}.

(a) For any x ∈ Sn \ {N }, show that σ(x) = u, where (u, 0) is the point where the line through N and x
intersects the linear subspace where xn+1 = 0. Similarly, show that (σ̃(x), 0) is the point where the line
through S and x intersects the same subspace. (For this reason, σ̃ is called stereographic projection
from the south pole.)
(b) Show that σ is bijective, and
2
(2u1 , . . . , 2un , |u| − 1)
σ −1 (u1 , . . . , un ) = 2 .
|u| + 1

(c) Compute the transition map σ̃ ◦ σ −1 and verify that the atlas consisting of the two charts (Sn \ {N }, σ)
and (Sn \ {S}, σ̃) defines a smooth structure on Sn . (The coordinates defined by σ and σ̃ are called
stereographic coordinates.)
(d) Show that this smooth structure is the same as the one defined in Example 1.31.

For (a), let x = (x1 , . . . , xn+1 ) ∈ Sn \ {N }. The line through N and x is parametrized by u1 =
x1 t, . . . , un = xn t, un+1 = (xn+1 − 1)t + 1. The intersection of this line with the un+1 = 0 occurs when

7
t = 1/(1 − xn+1 ). Hence, the intersection point is (σ(x), 0), as desired. Similarly, if x = (x1 , . . . , xn+1 ) ∈
Sn \ {S}, then the line through S and x is parametrized by u1 = x1 t, . . . , un = xn t, un+1 = (xn+1 + 1)t − 1.
Setting un+1 = 0, we find that the intersection point is (σ̃(x), 0).
For (b), it suffices to check that the given definition for σ −1 is indeed the inverse of σ. It is straightforward
2
to check that σ −1 (u) = 1 for all u ∈ Rn , and the last coordinate of σ −1 (u) is |u| −1

|u|2 +1
< 1. Hence,
σ −1 (Rn ) ⊆ Sn \{N }. Showing that σ ◦σ −1 and σ −1 ◦σ are the identity maps is a straightforward calculation.
For (c), the transition map is

(u1 , . . . , un )
σ̃ ◦ σ −1 (u1 , . . . , un ) = .
|u|

Note that the domain of σ̃ ◦ σ −1 is σ(Sn \ {N, S}). Since σ(S) = 0, this shows that u = 0 is not in the
domain of σ̃ ◦ σ −1 . Hence, σ̃ ◦ σ −1 is smooth. A similar computation shows that σ ◦ σ̃ −1 is smooth. Since
Sn \ {N } and Sn \ {S} form an open cover of Sn , we conclude that (Sn \ {N }, σ) and (Sn \ {S}, σ̃) define a
smooth structure on Sn .
+
For (d), there are three “types” of charts from Example 1.31 to consider. The first is (Un+1 , φ+
n+1 ),
− − ± ±
which contains N . The second is (Un+1 , φn+1 ), which contains S. Finally, the third type is (Ui , φi ) for
i = 1, . . . , n, which contain neither N nor S. We will show that each type of chart is smoothly compatible
with σ; the computation showing that they are smoothly compatible with σ̃ is similar. The first functions
for the first and second type of chart are

(2u1 , . . . , 2un )
φ±
n+1 ◦ σ
−1 1
(u , . . . , un ) = 2 ,
|u| + 1

which are both smooth. The inverses of these transition functions are
(u1 , . . . , un )
σ ◦ (φ±
n+1 )
−1 1
(u , . . . , un ) = q
2
1 ∓ 1 − |u|

Hence, σ◦(φ−
n+1 )
−1
is smooth. To see that σ◦(φ+n+1 )
−1
is smooth, note that its domain is (φ+ +
n+1 )(Un+1 \{N }),
which does not include 0. For the third type of chart, we have

ci , . . . , 2un , |u|2 − 1)
(2u1 , . . . , 2u
φ±
i ◦σ
−1 1
(u , . . . , un ) = 2 ,
|u| + 1

which is smooth, and


q
2
(u1 , . . . , ui−1 , 1 − |u| , ui , . . . , un−1 )
σ ◦ (φ±
i )
−1 1
(u , . . . , un ) = ,
1 − un

which is also smooth, since un 6= 1 in Ui± . Hence, the smooth atlas from Example 1.31 is smoothly compatible
with the smooth atlas described in this problem. Hence, they determine the same smooth structures on Sn .
Problem 1.8. By identifying R2 with C, we can think of the unit circle S1 as a subset of the complex plane.
An angle function on a subset U ⊆ S1 is a continuous function θ : U → R such that eiθ(z) = z for all
z ∈ U . Show that there exists an an angle function θ on an open subset U ⊆ S1 if and only if U 6= S1 .
For any such angle function, show that (U, θ) is a smooth coordinate chart for S1 with its standard smooth
structure.
(⇒) Let θ be an angle function on a subset U ⊆ S1 . For a contradiction, suppose that U = S1 . Since the
image of a connected and compact space under a continuous function is connected and compact, we have
that θ(S1 ) = [a, b] ⊆ R for some a ≤ b. We now show that θ̃ : S1 → [a, b], z 7→ θ(p) is a homeomorphism;
this will give us a contradiction, since [a, b] is simply connected, but S1 is not. It suffices to show that θ̃
is bijective, since a continuous bijection with compact domain has a continuous inverse. By definition, θ̃ is
0
surjective. If θ̃(z) = θ̃(z 0 ), then z = eiθ(z) = eiθ(z ) = z 0 , so θ̃ is injective, and therefore bijective.

8
(⇐) Let U ⊂ S1 be a proper open subset. Suppose that p ∈ S1 \ U . We will construct an angle function
θ on S1 \ {p}. The restriction of θ to U is therefore also an angle function. Note that θ(z) = −i ln(z) is
an angle function, where we must take a branch of the complex logarithm along the ray through p and the
origin.
Let θ : U → R be an angle function. We proved in that θ is injective in the ⇒ direction above. Hence,
by the theorem on invariance of domain, θ is an open map and therefore a homeomorphism onto its image.
−1
This shows that (U, θ) is a chart for S 1 . Suppose that U = S1 \ {N }. Since θ−1 (x) = eiθ(θ (x)) = eix
corresponds to (cos x, sin x) ⊆ Rn . Let σ be the stereographic projection from the north discussed in the
previous problem. Then
cos x x π
σ ◦ θ−1 (x) = = tan + ,
1 − sin x 2 4
which is a diffeomorphism from any interval of the form ( π2 + 2πn, 5π
2 + 2πn) to R, where n ∈ Z (proving
the above identity is a fun trigonometry exercise). Now, let U ⊂ S1 be an arbitrary proper open subset.
By rotating S1 , we may assume that N ∈ / U . Then σ ◦ θ−1 is the restriction of the above formula on each
connected component of U . Thus, (U, σ) is smoothly compatible with the stereographic coordinate charts,
and is thus a smooth coordinate chart for S1 with its standard smooth structure.
Problem 1.9. Complex projective n-space, denoted by CPn , is the set of all 1-dimensional complex-
linear subspaces of Cn+1 , with the quotient topology inhereted from the natural projection π : Cn+1 \ {0} →
CPn . Show that CPn is a compact 2n-dimensional topological manifold, and show how to give it a smooth
structure analogous to the one we constructed for RPn .
Let Uei ⊆ Cn+1 \ {0} be the open set of points where where z i 6= 0, and let Ui = π(U ei ). It is not too
−1 n+1
hard to show that π (π(Ui )) = Ui . Hence, Ui is open. Moreover, since the Ui cover C
e e e \ {0} and π is
surjective, the Ui form an open cover of CPn . Define φi : Ui → Cn ∼ = R2n by
 1
z i−1 z i+1 z n+1

1 n+1 z
φi [z , . . . , z ] = ,..., i , i ,..., i .
zi z z z

Since φi [z 1 , . . . , z n+1 ] is unchanged by multiplying (z 1 , . . . , z n+1 ) by a non-zero constant, φi is well-defined.


Moreover, φi is bijective, which is not too hard to see. We claim that π|Uei : U ei → Ui is a quotient map; to
show this, we must show that π is an open map, which was done in Exercise 1.6. Hence, by the characteristic
property of quotient maps, φi is continuous if and only if φi ◦ π|Uei is continuous. Since

z1 z i−1 z i+1 z n+1


 
φi ◦ π|Uei (z 1 , . . . , z n+1 ) = i
,..., i , i ,..., i ,
z z z z
φi is continuous. The inverse of φi is given by

φ−1 1 n 1
i (z , . . . , z ) = [z , . . . , z
i−1
, 1, z i , . . . , z n ].

Let fi : Cn → Cn+1 , (z 1 , . . . , z n ) 7→ (z 1 , . . . , z i−1 , 1, z i , . . . , z n ); note that fi is continuous. Then φ−1


i = π ◦ fi
is continuous. Thus φi is a homeomorphism, and we have shown that CPn is locally Euclidean.
To prove that CPn is Hausdorff, we will need the following fact from topology: if X is a compact Hausdorff
space and G is a compact topological group acting continuously on X, then the orbit space X/G is Hausdorff.
Let X = S2n+1 ⊆ Cn+1 and G = S1 ⊆ C, and let S1 × S2n+1 → S2n+1 : (λ, z) 7→ λz be the group action.
Then the quotient space S2n+1 /S1 is Hausdorff. We will now prove that S2n+1 /S1 is homeomorphic to CPn .
We will denote the equivalence classes in S2n+1 /S1 and CPn by [·]S2n+1 /S1 and [·]CPn , respectively. We claim
that the map i : S2n+1 /S1 → CPn , [z 1 , . . . , z n+1 ]S2n+1 /S1 7→ [z 1 , . . . , z n+1 ]CPn is a homeomorphism. The
fact that i is well-defined and bijective is easily verified. For continuity, observe the following commutative
diagram
i
S2n+1 C2n+1 \ {0}
π π

i
S2n+1 /S1 CPn

9
where i is the inclusion map, and the maps denoted π are the corresponding quotient maps. Commutativity
and the continuity of π ◦ i imply that i ◦ π is continuous. The universal property of quotient maps implies
that i is continuous. Since the inverse of a continuous function with compact domain is continuous, we have
that i is a homeomorphism. It follows that CPn ∼ = S2n+1 /S1 , and is therefore Hausdorff.
The proof that CP is second-countable is a straightforward adaptation of the proof that RPn is second-
n

countable given in Exercise 1.6.


We conclude the problem by showing that charts constructed above are smoothly compatible, and there-
fore define a smooth structure on CPn . Let (Ui , φi ) and (Uj , φj ) be two charts, with i > j. The transition
function is  1
z j−1 z j+1 z i−1 1 z i zn

−1 1 n z
φj ◦ φi (z , . . . , z ) = ,..., j , j ,..., j , j, j,..., j .
zj z z z z z z
Since z j 6= 0 for (z1 , . . . , z n ) ∈ φi (Ui ∩ Uj ), the above map is well defined, and therefore a diffeomorphism
from φi (Ui ∩ Uj ) to φj (Ui ∩ Uj ).
Problem 1.10. Let k and n be integers satisfying 0 < k < n, and let P, Q ⊆ Rn be the linear subspaces
spanned by (e1 , . . . , ek ) and (ek+1 , . . . , en ), respectively, where ei is the ith standard basis vector for Rn .
For any k-dimensional subspace S ⊆ Rn that has trivial intersection with Q, show that the coordinate
representation φ(S) constructed in Example 1.36 is the unique (n − k) × k matrix B such that S is spanned
by the columns of the matrix IBk , where Ik denotes the k × k identity matrix.
As described in Example 1.36, if πP : Rn → P and πQ : Rn → Q are the projections onto P and Q, then
the map πP |S : S → P is an isomorphism. Hence, there are vectors b1 , . . . , bk ∈ Q such that ei + qi ∈ S
for each i ∈ {1, . . . , k}. Since {e1 + b1 , . . . , ek + bk } gets mapped to the basis {e1 , . . . , ek } for P , it must be
a basis for S. Arranging the bi into columns, we have a (n − k) × k matrix B such that S is spanned by
the columns of IBk . Now let’s show that this B is unique. Suppose B 0 is a (n − k) × k matrix such that

Ik
form a basis for S. Since πP |S (ei + b0i ) = ei and πP |S is an isomorphism, we must have

the columns of B 0

bi = bi for each i. Hence, B = B 0 , so B is unique.


0

Since (πQ |S ) ◦ (πP |S )−1 (ei ) = (πQ |S )(ei + bi ) = bi , the matrix representation of (πQ |S ) ◦ (πP |S )−1 is B.
Hence, φ(S) = B.
Problem 1.11. Let M = Bn , the closed unit ball in Rn . Show that M is a topological manifold with
boundary in which each point in Sn−1 is a boundary point and each point in Bn is an interior point. Show
how to give it a smooth structure such that every smooth interior chart is a smooth chart for the standard
smooth structure on Bn .
Bn is Hausdorff and second-countable, since it is a subspace of Rn . Let Ui+ = {(x1 , . . . , xn ) ∈ Bn : xi > 0}
and Ui− = {(x1 , . . . , xn ) ∈ Bn : xi < 0}. Then the Ui± form an open cover of Bn . Define
q
± ± n,± 1 n 1 i−1 i i )2 − · · · − (xn )2 , xi+1 , . . . , xn ),
φi : Ui → Bi , (x , . . . , x ) 7→ (x , . . . , x , x ∓ 1 − (x1 )2 − · · · − (x
[

where Bn,+i = {(x1 , . . . , xn ) ∈ Bn : xi ≥ 0} and Bn,− i = {(x1 , . . . , xn ) ∈ Bn : xi ≤ 0}. The inverse of φ± i is


then
q
n,±
(φ±
i ) −1
: Bi → U i
±
, (x 1
, . . . , x n
) →
7 (x 1
, . . . , x i−1
, xi
± 1 − (x1 )2 − · · · − (x[ i )2 − · · · − (xn )2 , xi+1 , . . . , xn ).

Since both φ± i and (φi )


± −1
are continuous, we conclude that (Ui± , φ± i ) are boundary charts covering M , so
M is a topological manifold. q
Let (x1 , . . . , xn ) ∈ Sn−1 . Say (x1 , . . . , xn ) ∈ U ± . Then xi = ± 1 − (x1 )2 − · · · − (x
i
[ i )2 − · · · − (xn )2 .

Then φ± 1 n 1
i (x , . . . , x ) = (x , . . . , x
i−1
, 0, , xi+1 , . . . , xn ), so every point in Sn−1 is a boundary point. If
(x , . . . , x ) lies in B , then it lies in the interior chart (Bn , id), and is thus an interior point.
1 n n

Note that φ± ± −1
i and (φi ) can be extended to maps from R × · · · × R × (−1, 1) × R × · · · × R to itself,
where (−1, 1) is the ith factor, and . Since both extensions of φ± ± −1
i and (φi ) are both diffeomorphisms, the
transition functions are all smooth, so the charts given above define a smooth structure on M . Moreover,
since every chart is smoothly compatible with (Bn , id), every interior chart is smoothly compatible with the
standard smooth structure on Bn .

10
Problem 1.12. Prove Proposition 1.45 (a product of smooth manifolds together with one smooth manifold
with boundary is a smooth manifold with boundary).
We will show that if M is a smooth m-manifold and N is a smooth n-manifold with boundary, then M ×N
is a smooth manifold with boundary, and ∂(M × N ) = M × ∂N . The general case follows by induction.
First, M × N is Hausdorff and second-countable, since both M and N are. Given charts (U, φ) and (V, ψ)
for M and N , respectively, we let (U × V, φ × ψ) be a chart for M × N . The collection of all such charts then
gives M × N the structure of a smooth manifold with boundary, as we now show. Let (U1 × V1 , φ1 × ψ1 ) and
(U2 × V2 , φ2 × ψ2 ) be two intersecting charts. Note that (φ2 × ψ2 ) ◦ (φ1 × ψ1 )−1 = (φ2 ◦ φ−1 −1
1 ) × (ψ2 × ψ1 ) has
domain (φ1 ◦ ψ1 )([U1 × V1 ] ∩ [U2 × V2 ]) = φ1 (U1 ∩ U2 ) × ψ1 (V1 ∩ V2 ). Since ψ1 (V1 ∩ V2 ) can be extended to
an open set such that ψ2 ◦ ψ1−1 is smooth, we have that the product charts are smoothly compatible. This,
M × N is a smooth manifold with boundary.
Let (x, y) ∈ ∂(M × N ). Then (x, y) is in the domain of some boundary chart (U × V, φ × ψ). Since φ(U )
is open in Rm , it follows that (V, ψ) must be a boundary chart. Moreover, ψ(y) should lie on the boundary
of Hn , since otherwise there is an interior chart (V 0 , ψ 0 ) whose domain contains y 0 , and thus (U × V 0 , φ × ψ 0 )
is an interior chart containing (x, y 0 ). Thus, y ∈ ∂N , so (x, y) ∈ M × ∂N . Conversely, let (x, y) ∈ M × ∂N .
Then there is a boundary chart (V, ψ) such that y ∈ V and ψ(y) is in ∂Hn . Then, if u is in the domain of a
chart (U, φ), then (φ×ψ)(x, y) = (φ(x), ψ(y)) ∈ ∂Hm+n , so (x, y) ∈ ∂(M ×N ). Hence, ∂(M ×N ) = M ×∂N .

2 Smooth Maps
Exercise 2.1. Let M be a smooth manifold with or without boundary. Show that pointwise multiplication
turns C ∞ (M ) into a commutative ring and a commutative and associative algebra over R.
Let f, g ∈ C ∞ (M ). We want to show that f + g and f g are also smooth functions. By Exercise 2.3,
f ◦ φ−1 and g ◦ φ−1 are smooth for every coordinate chart (U, φ). Then (f + g) ◦ φ−1 = (f ◦ φ−1 ) + (g ◦ φ−1 )
and (f g) ◦ φ−1 = (f ◦ φ−1 )(g ◦ φ−1 ) are smooth. Thus C ∞ (M ) is a commutative ring, since multiplication in
R is commutative. Since smooth functions can be multiplied by real constants without affecting smoothness,
C ∞ (M ) is also a commutative and associative algebra over R, since multiplication of functions is commutative
and associative.
Exercise 2.2. Let U be an open submanifold of Rn with its standard smooth manifold structure. Show that
a function f : U → Rk is smooth in the sense just defined if and only if it is smooth in the sense of ordinary
calculus. Do the same for an open submanifold with boundary in Hn .
The chart (U, id) defines a smooth structure on U . Let f be smooth in the sense defined in the section.
Then, for each chart p ∈ U , there is a chart (V, ψ) whose domain contains p and such that f ◦ ψ −1 is smooth.
Since ψ ◦ id−1 = ψ is smooth, we have that (f ◦ ψ −1 ) ◦ ψ = f is smooth on V . Since this holds for every
chart, we conclude that f is smooth in the ordinary calculus sense. Now suppose that f is smooth in the
sense of ordinary calculus. Then, since every p ∈ U and f ◦ id−1 is smooth, f is smooth in the sense of the
section.
By Exercise 1.44, if U ⊆ Hn is open, then (U, id) gives U a smooth manifold structure, since id : U → U
can be extended to all of R. Let f : U → R be smooth in the sense of the section. Then for every p ∈ M ,
there is an open set V 3 p in U and a chart (V, ψ) such that f ◦ ψ −1 is smooth on some open extension of
V . Since ψ ◦ id−1 = ψ is also smooth on some open extension of V , we conclude that f is smooth on some
open extension of V , in the sense of ordinary calculus. Conversely, let f : U → R be smooth in the sense
of ordinary calculus. Then f = f ◦ id−1 is smooth on some open extension of U . Then f is smooth in the
sense defined in the section.
Exercise 2.3. Let M be a smooth manifold with or without boundary, and suppose f : M → Rk is a smooth
function. Show that f ◦ φ−1 : φ(U ) → Rk is smooth for every smooth chart (U, φ).
Let (U, φ) be an arbitrary chart, and let p ∈ U . Since f is smooth, there is a chart (V, ψ) such that p ∈ V
and f ◦ ψ −1 : ψ(V ) → Rk is smooth. Since the transition function ψ ◦ φ−1 : φ(U ∩ V ) → ψ(U ∩ V ) is smooth,
we have that (f ◦ ψ −1 ) ◦ (ψ ◦ φ−1 ) = f ◦ φ−1 : φ(U ∩ V ) → Rk is smooth. Since p ∈ U was chosen arbitrarily,
we conclude that f ◦ φ−1 is smooth on all of φ(U ).

11
Exercise 2.7. Prove propositions 2.5 and 2.6.
We begin with Proposition 2.5.
(Smoothness ⇒ (b)) By smoothness of F , there exists smooth atlases {(Uα , φα )} and {(Vβ , ψβ )} for M
and N , respectively, such that for every α there exists a β such that F (Uα ) ⊆ Vβ and ψβ ◦ F ◦ φ−1 α is smooth.
Now, let (Uα , φα ) and (Vβ , ψβ ) be arbitrary charts in the atlases described above. Let (Vβ 0 , ψβ 0 ) be a chart
−1
such that F (Uα ) ⊆ Vβ 0 . We have that ψβ 0 ◦ F ◦ φ−1 α is smooth from φα (Uα ) to ψβ 0 (Vβ 0 ), and ψβ ◦ ψβ 0 is
smooth from ψβ 0 (Vβ ∩ Vβ 0 ) to ψβ (Vβ ∩ Vβ 0 ). Hence, (ψβ ◦ ψβ−1 −1
0 ) ◦ (ψβ 0 ◦ F ◦ φα ) = ψβ ◦ F ◦ φα
−1
is smooth
−1
from φα (Uα ∩ F (Vβ )) to ψβ (Vβ ) (there is no issue with extending the codomain here).
((b) ⇒ (a)) Since {(Uα , φα )} and {(Vβ , ψβ )}, for every p ∈ M , p ∈ Uα for some α and F (p) ∈ Vβ for
some β. Let U = Uα and V = Vα . By continuity of F , U ∩ F −1 (V ) is open. By assumption, ψ ◦ F ◦ φ−1 :
φ(U ∩ F −1 (V )) → ψ(V ) is smooth.
((a) ⇒ Smoothness) Let p be in M , and let (U b , φ) and (V, ψ) be charts containing p and F (p), respectively.
−1
Taking U = U ∩ F (V ), we find that F is smooth.
b
We turn to Proposition 2.6.
For (a), suppose that every p ∈ M has a neighbourhood U such that F |U is smooth. Then there is a
chart (U e , φ̃) of U containing p and a chart (V, ψ) of N containing F (p) such that F (U e ) ⊆ V and ψ ◦ F ◦ φ−1
is smooth. Since U e is open in U and U is open in M , we have that (U e , φ̃) is a chart of M containing p. Thus
F is smooth.
For (b), let U ⊆ M be an open submanifold with its standard smooth structure. Let p ∈ U and let
(U 0 , φ) and (V, ψ) be charts containing p and F (p) such that ψ ◦ F ◦ φ−1 is smooth. Then the restriction
(U ∩ U 0 , φ) is a chart in U having all the required properties for smoothness. Thus, F |U is smooth.
Exercise 2.9. Suppose F : M → N is a smooth map between smooth manifolds with or without boundary.
Show that the coordinate representation of F with respect to every pair of smooth charts for M and N is
smooth.
Let (U, φ) and (V, ψ) be an arbitrary pair of charts for M and N , respectively. If U ∩ F −1 (V ) = ∅, then
there is nothing to check, so suppose that p ∈ U ∩ F −1 (V ), so that F (p) ∈ V . Since F is smooth, there is a
chart (U e ) ⊆ Ve and ψ̃ ◦ F ◦ φ̃−1 : φ̃(U
e , φ̃) containing p and a chart (Ve , ψ̃) such that F (U e ) → ψ̃(Ve ) is smooth.
−1 −1
Since φ̃ ◦ φ : φ(U ∩ U ) → φ̃(U ∩ U ) and ψ ◦ ψ̃ : ψ̃(V ∩ V ) → ψ(V ∩ V ) are smooth, it follows that
e e e e

(ψ ◦ ψ̃ −1 ) ◦ (ψ̃ ◦ F ◦ φ̃−1 ) ◦ (φ̃ ◦ φ−1 ) = ψ ◦ F ◦ φ−1 : φ(U ∩ U


e ∩ F −1 (V )) → ψ(V ).

Since p ∈ U ∩ F −1 (V ) was arbitrary, we conclude that ψ ◦ F ◦ φ−1 is smooth on all of φ(U ∩ F −1 (V )).
Exercise 2.11. Prove parts (a)-(c) of Proposition 2.10.
For (a), let c : M → N, p 7→ c be a constant map, where c is some fixed point in N . Let p ∈ M be arbitrary,
and let (U, φ) be a chart containing p and (V, ψ) a chart containing F (p) = c. Since U ∩F −1 (V ) = U ∩M = U
is open and ψ ◦ F ◦ φ−1 : φ(U ) → ψ(V ), x 7→ ψ(c) is smooth (as a constant map between Euclidean spaces),
we have that c : M → N is smooth.
For (b), let id : M → M be the identity map. Let p ∈ M and let (U, φ) be a chart containing p. Then
(U, φ) is a chart containing id(p) = p, and φ ◦ id ◦ φ−1 : φ(U ) → φ(U ) is smooth since it is the identity map,
and the identity map on a subset of Euclidean space is smooth.
For (c), let U ⊆ M be an open submanifold and let i : U ,→ M be the inclusion map. Let p ∈ U
and let (V, φ) be a chart in U containing p. Then (V, φ) is also a chart in M containing i(p) = p. Then,
φ ◦ i ◦ φ−1 : φ(U ) → φ(U ) is the identity map, which is smooth. Hence i : U ,→ M is smooth.
Exercise 2.16. Prove Proposition 2.15.
For (a), let F : M → N and G : N → P be diffeomorphisms. We that G ◦ F : M → P is a bijection and
is smooth by Proposition 2.10. Since F −1 and G−1 are also smooth, the composition F −1 ◦ G−1 = (G ◦ F )−1
is also smooth by Proposition 2.10. We conclude that G ◦ F is a diffeomorphism.
For (b), let Fi : Mi → Ni be diffeomorphisms between smooth manifolds for i = 1, . . . , n. We want to
show that
F1 × · · · × Fn : M1 × · · · × Mn → N1 × · · · × Nn , (p1 , . . . , pn ) 7→ (F1 (p1 ), . . . , Fn (pn ))

12
is a diffeomorphism. The inverse of F1 × · · · × Fn is F1−1 × · · · × Fn−1 . By Proposition 2.12, F = F1 × · · · × Fn
is smooth if and only if each πi ◦ F is smooth for each i, where πi is the projection from N1 × · · · Nn 7→ Ni .
Since (πi ◦ F )(p1 , . . . , pn ) = Fi (pi ), we have that πi ◦ F = Fi ◦ πi , where the second πi is the projection from
M1 × · · · × Mn to Mi . Since Fi and πi are smooth, πi ◦ F is smooth, and therefore F is smooth. A similar
proof shows that F −1 is smooth.
For (c), let F : M → N be a diffeomorphism. Then F and F −1 are continuous by Proposition 2.4, so F
is a homeomorphism. Then F is and open map since homeomorphisms are open maps.
For (d), let F : M → N be a diffeomorphism, let U ⊆ M be an open submanifold, and let F |U : U → F (U )
be the restriction. Note that F |U = F ◦ i, where i : U ,→ M is the inclusion. By Exercise 2.11, F |U is
smooth. The same argument applies to (F |U )−1 : F (U ) → U , since F (U ) is an open submanifold by part
(c).
For (e), M is always diffeomorphic to M , since id : M → M is a diffeomorphism; being diffeomorphic
is reflexive. If F is a diffeomorphism, then so is F −1 ; being diffeomorphic is symmetric. Finally, since
the composition of diffeomorphisms is a diffeomorphism, being diffeomorphic is transitive. Hence, being
diffeomorphic is an equivalence relation.
Exercise 2.19. Use Theorem 1.46 to prove Theorem 2.18.
Let F (p) ∈ F (∂M ). Then p ∈ ∂M , so there is a boundary chart (U, φ) containing p. Then (F (U ), φ◦F −1 )
is a boundary chart, since F and φ are diffeomorphisms and thus homeomorphisms. Hence, F (p) ∈ ∂N .
Hence, F (∂M ) ⊆ ∂N ; applying F −1 to both sides, we have ∂M ⊆ F −1 (∂N ). But F −1 is a diffeomorphism,
so the same argument given above shows that F −1 (∂N ) ⊆ ∂M . Thus F (∂M ) = ∂N .
Since IntM is open by Exercise 1.39, F restricts to a diffeomorphism from IntM to F (IntM ) by Propo-
sition 2.15. Since F (∂M ) = ∂N , and every point is either an interior point or a boundary point, we have
that F (IntM ) = IntN .
Exercise 2.24. Show how the proof of Theorem 2.23 needs to be modified for the case in which M has
nonempty boundary.
We need our bases to be composed of regular balls and half balls. We also need to extend our definition
of bump functions to include coordinate half balls. Otherwise the proof is essentially the same.
Exercise 2.27. Give a counterexample to show that the conclusion of the extension lemma can be false if
A is not closed.
Let M = R (with its standard smooth structure) and let A = (−∞, 0) ∪ (0, ∞). Let f : A → R be defined
by f (x) = 1 if x > 0 and f (x) = −1 if x < 0. Note that f is smooth since it is smooth on each component
of A. Any extension of f to R is discontinuous, and therefore not smooth, so the extension lemma does not
hold here.
Problem 2.1. Define f : R → R by (
1, x ≥ 0,
f (x) =
0, x < 0.
Show that for every x ∈ R, there are smooth coordinate charts (U, φ) containing x and (V, ψ) containing f (x)
such that ψ ◦ f ◦ φ−1 is smooth as a map from φ(U ∩ f −1 (V )) to ψ(V ), but f is not smooth in the sense we
have defined in this chapter.
Since f is not continuous, it is not smooth in the sense we have defined in this chapter. Moreover, f is
smooth away from x = 0 since it is constant there. Let  > 0, let U = (−, ), and let V = ( 21 , 32 ). Then
U contains x = 0 and V contains f (x) = 1. Moreover, we let (U, id) and (V, id) be charts for R. Then
id ◦ f ◦ id−1 = f is the constant map on f (U ∩ f −1 (V )) = [0, ), and is therefore smooth.
Problem 2.2. Prove Proposition 2.12.
Let F : N → M1 × · · · × Mk be smooth. By Proposition 2.10, in order to show that πi ◦ F is smooth, it
suffices to show that the projection πi : M1 × · · · × Mk → Mi is smooth. Let (p1 , . . . , pk ) ∈ M1 × · · · × Mk ,
and let (U1 × · · · × Uk , φ1 × · · · × φk ) be a product chart containing (p1 , . . . , pk ). Then (Ui , φi ) is a chart

13
containing pi ∈ Mi . The coordinate representation of φi ◦ πi ◦ (φ1 × · · · × φk )−1 is just the projection from
φ1 (U1 ) × · · · × φk (Uk ) to φi (Ui ), which is smooth. Hence, πi is smooth.
Conversely, let πi ◦ F be smooth for each i = 1, . . . , k. Let p ∈ N , let (V, ψ) be a chart containing p and
let (Ui , φi ) be charts containing πi ◦ F (p) such that πi ◦ F (V ) ⊆ Ui and φi ◦ πi ◦ F ◦ ψ −1 : ψ(V ) → φi (Ui )
is smooth for each i = 1, . . . , k. Now, then (U1 × · · · × Uk , φ1 × · · · × φk ) is a chart for M1 × · · · × Mk
such that F (V ) ⊆ U1 × · · · × Uk . Then the coordinate representation (φ1 × · · · × φk ) ◦ F ◦ ψ −1 : ψ(V ) →
φ1 (U1 ) × · · · × φk (Uk ) is smooth, since the projection onto each φi (Ui ) is equal to φi ◦ πi ◦ F ◦ ψ −1 , which is
smooth.
Problem 2.3. For each of the following maps between spheres, compute sufficiently many coordinate repre-
sentations to prove that it is smooth.
(a) pn : S1 → S1 is the nth power map for n ∈ Z, given in complex notation by pn (z) = z n .
(b) α : Sn → Sn is the antipodal map α(x) = −x.
(c) F : S3 → S2 is given by F (w, z) = (z w̄ + wz̄, iwz̄ − iz w̄, z z̄ − ww̄), where we think of S3 as the subset
2 2
{(w, z) : |w| + |z| = 1} of C2 .
For (a), let z ∈ S1 , let (U, θ) be an angle coordinate chart containing z, and let (V, φ) be an angle
coordinate chart containing z n . Then φ ◦ pn ◦ θ−1 (x) = φ ◦ pn (eix ) = φ(einx ) = nx + 2kπ for some k, which is
constant on each component of θ(U ∩ p−1 −1
n (V )). Note that U ∩ pn (V ) is open, since pn is continuous. Hence,
pn is smooth.
For (b), let p ∈ Sn , and assume that (Sn \{N }, σ) is the stereographic chart from the north and it contains
p. Then (Sn \ S, σ̃) contains α(p), where σ̃ is the stereographic projection from the south. A computation
shows that σ̃ ◦ α ◦ σ −1 (u) = −u, which is smooth. Hence, α is smooth.
For (c), writing F in real coordinates, we have

F (x1 , x2 , x3 , x4 ) = (2x1 x3 + 2x2 x4 , 2x1 x4 − 2x2 x3 , (x3 )2 + (x4 )2 − (x1 )2 − (x2 )2 ),

so F is continuous, since it is the restriction of a continuous function. A computation shows that


2 2
(8u1 u3 + 4u2 (|u| − 1), 4u1 (|u| − 1) − 8u2 u3 )
σS2 ◦ F ◦ σS−1
3 (u1 , u2 , u3 ) = 2 ,
1 + (2u1 )2 + (2u2 )2 − (2u3 )2 − (|u| − 1)2
which is smooth on σS3 (S3 \ {N } ∩ F −1 (S2 \ {N })). Here σSn is the stereographic projection from the north
of Sn . Similar computations for different pairs of charts show that F is indeed a smooth function.
Problem 2.4. Show that the inclusion map Bn ,→ Rn is smooth when Bn is regarded as a smooth manifold
with boundary.
We give Bn the smooth structure described in 1.11. Let x ∈ Bn be contained in some chart (Ui± , φ± i ). Then
i(x) = x is contained in the chart (Rn , id), where i : Bn ,→ Rn is the inclusion. Then id ◦i◦(φ±
i ) −1
= (φ±i )
−1
,
which is smooth, as discussed in Problem 1.11.
Problem 2.5. Let R be the real line with its standard smooth structure, and let R
e denote the same topological
manifold with the smooth structure defined in Example 1.23. Let f : R → R be a function that is smooth in
the usual sense.
(a) Show that f is also smooth as a map from R to R.
e
(b) Show that f is smooth as a map from Re to R if and only if f (n) (0) = 0 whenever n is not an integral
multiple of 3.
For (a), the coordinate representation ψ ◦ f ◦ id−1 = ψ ◦ f is smooth, since ψ(x) = x3 and f are both
smooth in the sense of regular calculus.
For (b), the coordinate representation is id ◦ f ◦ ψ −1 = f ◦ ψ −1 , and f ◦ ψ −1 (x) = f (x1/3 ). Let f ◦ ψ −1 .
We appeal to Faà di Bruno’s formula, which states that for smooth functions F and G
n  (j) mj
dn X n! (m1 +···+mn )
Y G (x)
F (G(x)) = F (G(x)) ,
dxn m1 ! · · · mn ! j=1
j!

14
where the sum ranges over all n-tuples of nonnegative integers (m1 , . . . , mn ) such that m1 +2m2 +· · ·+nmn =
n. Since we are assuming f ◦ ψ −1 is smooth, we can apply the above formula to F = f ◦ ψ −1 and G = ψ.
Note that ψ (j) (0) = 0 for all j 6= 3. Hence, if f (n) (0) 6= 0, then there must be an n-tuple (m1 , . . . , mn ) such
that mj = 0 for all j 6= 3. Then n = 3m3 . This proves the first direction.
For the other direction, we first prove the following fact by induction on k: if F ∈ C k (R), then so is
k+ 31 1 1 1
x F (x 3 ). For k = 0, the result is clear, since xk+ 3 F (x 3 ) is continuous. For the inductive step, let
k
F ∈ C (R). Then
d h k+ 1 1
i 1 1 1 1 1 1
x 3 F (x 3 ) = x(k−1)+ 3 F (x 3 ) + x(k−1)+ 3 x 3 F 0 (x 3 ).
dx 3
1 1 1 1
By induction, x(k−1)+ 3 F (x 3 ) ∈ C k−1 (R). Since xF 0 (x) ∈ C k−1 (R), we also have 31 x(k−1)+ 3 F 0 (x 3 ) ∈
1 1
C k−1 (R) by induction. Since it derivative is C k−1 (R), we have that xk+ 3 F (x 3 ) ∈ C k (R), concluding the
proof of the claim.
Suppose that f (n) (0) = 0 for all non-multiples of 3. Using the Taylor remainder theorem to write f up
to the 3mth term:
f (3) (0) 3 f (3m) (0) 3m
f (x) = f (0) + x + ··· + x + x3m+1 F3m+1 (x)
3! (3m)!
for some smooth function F3m+1 . Then

1 f (3) (0) f (3m) (0) m 1 1


f (x 3 ) = f (0) + x + ··· + x + xm+ 3 F3m+1 (x 3 ).
3! (3m)!
1 1
By the claim, xm+ 3 F3m+1 (x 3 ) ∈ C m (R). Hence, f ◦ ψ −1 ∈ C m (R) for all m ≥ 0, and is therefore smooth.
Problem 2.6. Let P : Rk+1 \ {0} → Rk+1 \ {0} be a smooth function, and suppose that for some d ∈ Z,
P (λx) = λd P (x) for all λ ∈ R \ {0} and x ∈ Rn+1 \ {0}. (Such a function is said to be homogenous of
degree d.) Show that the map Pe : RPk → RPk defined by Pe([x]) = [P (x)] is well defined and smooth.

If [x] = [y], then x = λy for some λ ∈ R \ {0}. Then Pe([x]) = [P (x)] = [P (λy)] = [λd P (y)] = [P (y)] =
Pe([y]). Therefore, Pe is well defined.
Let [x] ∈ Ui and suppose that Pe([x]) ∈ Uj . The coordinate representation φj ◦ Pe ◦ φ−1i takes a point
(u , . . . , uk ) ∈ φi (Ui ) to
1

(P1 (α), . . . , Pj−1 (α), Pj+1 (α), . . . , Pn+1 (α))


,
Pj (α)
where α = φi (u1 , . . . , uk ) = (u1 , . . . , ui−1 , 1, ui , . . . , uk ) and Pm is the mth component of P . Since P is
smooth, each Pm is smooth. Therefore, the coordinate representation is smooth, and thus so is Pe.
Problem 2.7. Let M be a nonempty smooth n-manifold with or without boundary, and suppose n ≥ 1.
Show that the vector space C ∞ (M ) is infinite-dimensional.
We begin by proving the suggested claim from the hint, namely that if f1 , . . . , fk ∈ C ∞ (M ) have disjoint
support and are nonzero, then they are linearly independent. Consider a linear combination of these functions
set to 0:
α1 f1 + · · · + αk fk = 0.
For each i = 1, . . . , k, choose some xi ∈ M such that fi (xi ) 6= 0. Since fj (xi ) = 0 for each j 6= i, we have
αi f (xi ) = 0 ⇒ αi = 0. Since i was arbitrary, the fi are linearly independent.
Now, let (U, φ) be some chart and let k ∈ N be arbitrary. It is not hard to see that there are k disjoint
open balls Bri (φ(xi )) in φ(U ). By Proposition 2.25, there are smooth bump functions that are positive
on φ−1 (Bri /2 (φ(xi ))) and supported on φ(Bri (φ(xi ))). Since the support of all these balls are disjoint, the
corresponding bump functions are linearly independent. Since k ∈ N was arbitrarily chosen, we have that
C ∞ (M ) is infinite dimensional.
Problem 2.8. Define F : Rn → RPn by F (x1 , . . . , xn ) = [x1 , . . . , xn , 1]. Show that F is a diffeomorphism
onto a dense open subset of RPn . Do the same for G : Cn → CPn defined by G(z 1 , . . . , z n ) = [z 1 , . . . , z n , 1].

15
We claim that F : Rn → Un+1 is a diffeomorphism, where Un+1 is an open submanifold of RPn . We
begin by showing that F is injective; suppose that [x1 , . . . , xn , 1] = [y 1 , . . . , y n , 1]. Then (x1 , . . . , xn , 1) =
λ(y 1 , . . . , y n , 1), so 1 = λ · 1 ⇒ λ = 1. To see that F is surjective, let [x1 , . . . , xn+1 ] ∈ Un+1 . Then
x1 n
F ( xn+1 , . . . , xxn+1 ) = [x1 , . . . , xn+1 ]. The coordinate representation of F is

φn+1 ◦ F : Rn → φn+1 (Un+1 ), (x1 , . . . , xn ) 7→ (x1 , . . . , xn ),

which is of course smooth. The inverse of F is given by

x1 xn
 
−1 n 1 n+1
F : Un+1 → R , [x , . . . , x ] 7→ , . . . , .
xn+1 xn+1

The coordinate representation is F −1 ◦ φ−1 n+1 = (φn+1 ◦ F )


−1
= id which is smooth. Hence, F is a diffeomor-
phism. It remains to show that Un+1 is open and dense in RPn . We know that Un+1 is open since it is the
domain of a chart of RPn .
For density, let [x1 , . . . , xn+1 ] ∈ RPn \ Un+1 and let U ⊆ RPn be an open set containing [x1 , . . . , xn+1 ].
Since π is surjective, we have that U = π(π −1 (U )). Since π is continuous, we have that π −1 (U ) is open, and
therefore it contains a point (y 1 , . . . , y n+1 ) such that y n+1 6= 0. Hence, [y 1 , . . . , y n+1 ] ∈ U ∩ Un+1 . Thus,
every open set containing [x1 , . . . , xn+1 ] intersects Un+1 , and therefore Un+1 is dense in RPn .
Doing the same for G : C → CPn is completely analogous.

Problem 2.9. Given a polynomial p in one variable with complex coefficients, not identically zero, show
that there is a unique smooth map pe : CP1 → CP1 that makes the following diagram commute, where CP1 is
1-dimensional complex projective space and G : C → CP1 is the map from Problem 2.8:
G
C CP1
p p̃

G
C CP1

Let p(z) = an z n + · · · + a1 z + a0 with an 6= 0. Define P : C2 \ {0} → C2 \ {0} by

P (z 1 , z 2 ) = (an (z 1 )n + an−1 (z 1 )n−1 z 2 + · · · + a1 z 1 (z 2 )n−1 + a0 (z 2 )n , (z 2 )n ).

To see that the image of this map does not include 0, not that if z 2 = 0, then z1 6= 0 and P (z 1 , 0) =
(an (z 1 )n , 0) 6= 0. Since P is homogeneous of degree n, p̃ : CP1 → CP1 , [z 1 , z 2 ] 7→ [P (z 1 , z 2 )] is well defined.
It follows directly from the definition that p̃ ◦ G = G ◦ p. Since P is smooth, we have that p̃ is smooth by
Problem 2.6. If there is another smooth p̃0 making the diagram commute, then p̃0 must agree with p̃ on the
image of G, which is dense in CP1 . Since p̃ and p̃0 are smooth, they are equal.

Problem 2.10. For any topological space M , let C(M ) denote the algebra of continuous functions f : M →
R. Given a continuous map F : M → N , define F ∗ : C(N ) → C(M ) by F ∗ (f ) = f ◦ F .
(a) Show that F ∗ is a linear map.
(b) Suppose M and N are smooth manifolds. Show that F : M → N is smooth if and only if F ∗ (C ∞ (N )) ⊆
C ∞ (M ).
(c) Suppose F : M → N is a homeomorphism between smooth manifolds. Show that it is a diffeomorphism
if and only if F ∗ restricts to an isomorphism from C ∞ (N ) to C ∞ (M ).
For (a), let f, g ∈ C ∞ (M ) and let α, β ∈ R. Then F ∗ (αf + βg) = (αf + βg) ◦ F = αf ◦ F + βg ◦ F .
Hence, F ∗ is linear.
For (b), let F : M → N be smooth, and let f ∈ C ∞ (N ). Since the composition of smooth functions is
smooth, F ∗ (f ) = f ◦ F ∈ C ∞ (M ). Hence, F ∗ (C ∞ (N )) ⊆ C ∞ (M ). Conversely, suppose that F ∗ (C ∞ (N )) ⊆
C ∞ (M ). Let B = {(V, ψ)} be an atlas of regular coordinate balls in N and let A = {(U, φ)} any atlas for
M . By the extension lemma for smooth functions (Lemma 2.26), each coordinate map ψ : V → Rn can be
extended to a smooth map ψ : N → Rn , note that each extended component function ψ i : N → R is in

16
C ∞ (N ). Let p ∈ M be arbitrary and be contained in a smooth chart (U, φ), and let F (p) be in the chart
(V, ψ). We want to show that ψ ◦ F ◦ φ−1 : φ(U ∩ F −1 (V )) → ψ(V ) is smooth. Note that U ∩ F −1 (V ) is open
since F is assumed to be continuous. We have that ψ ◦ F : M → Rn is smooth, since each of the component
functions are F ∗ (ψ i ) ∈ C ∞ (M ). The restriction to U ∩ F −1 (V ) is also smooth, and since φ−1 is smooth, we
have that ψ ◦ F ◦ φ−1 is smooth. Hence, F is smooth.
For (c), let F be a diffeomorphism. Then F ∗ (C ∞ (N )) ⊆ C ∞ (M ) and (F −1 )∗ (C ∞ (M )) ⊆ C ∞ (N ). Since
F is invertible, F ∗ must be injective. We conclude that F ∗ : C ∞ (N ) → C ∞ (M ) is a bijection. It is also
linear by part (a). Finally, if f, g ∈ C ∞ (N ), then F ∗ (f g) = (f g) ◦ F = (f ◦ F )(g ◦ G). Thus, F ∗ is an
algebra isomorphism when restricted to C ∞ (N ). Conversely, suppose that F ∗ restricts to an isomorphism
from C ∞ (N ) to C ∞ (M ). Then F ∗ (C ∞ (N )) ⊆ C ∞ (M ) and (F −1 )∗ (C ∞ (M )) ⊆ C ∞ (N ), so part (b) implies
that F and F −1 are smooth, and therefore that F is a diffeomorphism.

Problem 2.11. Suppose V is a real vector space of dimension n ≥ 1. Define the projectivization of V ,
denoted by P(V ), to be the set of 1-dimensional linear subspaces of V , with the quotient topology induced by
the map π : V \ {0} → P(V ) that sends x to its span. (Thus P(Rn ) = RPn .) Show that P(V ) is a topological
(n − 1)-manifold, and has a unique smooth structure with the property that for each basis (E1 , . . . , En ) for V ,
the map E : RPn−1 → P(V ) defined by E[v 1 , . . . , v n ] = [v i Ei ] (where the brackets denote equivalence classes)
is a diffeomorphism.
Let V and W be n-dimensional vector spaces, and let T : V → W be a linear homeomorphism (V and W
have topologies inherited from any of their norms and the topologies are independent of the chosen norms).
Define Te : P(V ) → P(W ), [v] 7→ [T (v)]. Linearity of T guarantees that Te is well defined. If [T (v1 )] = [T (v2 )],
then T (v1 ) = λT (v2 ) = T (λv2 ) ⇒ v1 = λv2 ⇒ [v1 ] = [v2 ], so Te is injective; Te is clearly surjective, so it is
bijective.
We want to show that Te is a homeomorphism. Let πV : V → P(V ) and πW : W → P(W ) be the natural
projections, and let U ⊆ P(W ) be open. Te−1 (U ) is open if and only πV−1 (Te−1 (U )) = (Te ◦ πV )−1 (U ) is. But
Te ◦ πV = πW ◦ T , which is continuous, hence πV−1 (Te−1 (U )) is open, so Te−1 (U ) is open. This show that Te
is continuous. The same idea shows that Te−1 (which is given by Te−1 (w) = [T −1 (w)]) is continuous, since
T −1 is also a linear homeomorphism. We conclude that P(V ) and P(W ) are homeomorphic. Since V is
homeomorphic to Rn via a linear homeomorphism taking any basis of V to the standard basis of Rn , we
have that P(V ) ∼ = P(Rn ) = RPn−1 . Since RPn−1 is a topological (n − 1)-manifold, P(V ) is too.
Fix a basis (B1 , . . . , Bn ) for V , and let T : Rn → V be the linear homeomorphism that sends the standard
basis vector ei to Bi for each i = 1, . . . , n. Let Te : RPn−1 → P(V ) be the induced homeomorphism. Consider
the standard smooth atlas {(Ui , φi )}ni=1 for RPn−1 . I claim that {(Te(Ui ), φi ◦ Te−1 } is a smooth atlas for
P(V ). Indeed, the transition functions are (φi ◦ Te−1 ) ◦ (φj ◦ Te−1 )−1 = φi ◦ φ−1 j which are smooth. Now
n−1
let (E1 , . . . , En ) be a basis for V , and let E : RP → P(V ) be defined as above. Then the coordinate
representation of E is

(φi ◦ Te−1 ) ◦ E ◦ φ−1 1


j (x , . . . , x
n−1
) = φi [T −1 F (x1 , . . . , xj−1 , 1, xj , . . . , xn−1 )],

where F : Rn → V is defined by F (v 1 , . . . , v n ) = v i Ei . Since T −1 F is an invertible linear map from Rn to


itself, it is a diffeomorphism. Since φi and π : Rn → RPn−1 are smooth, the coordinate representation is
smooth. Therefore, E is smooth. The coordinate representation of E −1 is

φi ◦ E −1 ◦ (φj ◦ Te−1 )−1 (x1 , . . . , xn−1 ) = φi [F T −1 (x1 , . . . , xj−1 , 1, xj , . . . , xn−1 )],

so E −1 is smooth as well. Therefore, E is a diffeomorphism.


Let A be a smooth atlas for P(V ) such that Te is a diffeomorphism, and let (U, φ) ∈ A be a smooth chart.
Then φi ◦ Te−1 ◦ φ−1 is smooth for each i. But this just means that A is smoothly compatible with the smooth
structure described in this section. This proves uniqueness of the smooth structure.

Problem 2.12. State and prove the analogue of Problem 2.11 for complex vector spaces.
If V is a complex vector space of dimension ≥ 1, define the projectivization of V , denoted by P(V ) to be
the set of all 1-dimensional complex linear subspaces of V , with the quotient topology induced by the map

17
π : V \ {0} → P(V ) that sends x to its span. Then P(V ) is a topological (2n − 2)-manifold, and has a unique
smooth structure with the property that for each basis (E1 , . . . , En ) fpr V , the map E : CPn−1 → P(V )
defined by E[z 1 , . . . , z n ] = [z i Ei ] is a diffeomorphism.
The proof of the previous problem only used the fact that all n-dimensional real vector spaces are
isomorphic (and homeomorphic) via a linear map that sends bases to bases, as well as the structure of the
coordinate charts on RPn−1 . Since the same fact is true of complex vector spaces, and the coordinate charts
on CPn−1 are defined completely analogously to the coordinate charts on RPn−1 , the proof of the above
statement is given in the previous problem, with the exception that we must replace R with C whenever it
appears.
Problem 2.13. Suppose that M is a topological space with the property that for every indexed open cover
X of M , there exists a partition of unity subordinate to X. Show that M is paracompact.
Let X = {Xα }α be an open cover, and let (ψα )α be a partition of unity subordinatePto it. Let Uα =
{p ∈ M : ψα (p) > 0} = ψα−1 ((0, ∞)). Then Uα is open, since ψα is continuous. Since α ψα (p) = 1 for
all p ∈ M , we have that the Uα cover M . Moreover, since Uα ⊆ supp(ψα ) and {sup(ψα )} is locally finite,
{Uα }α is locally finite. Moreover, Uα ⊆ Xα , so {Uα }α is a refinement of X. Hence, M is paracompact.

Problem 2.14. Suppose A and B are disjoint closed subsets of a smooth manifold M . Show that there
exists f ∈ C ∞ (M ) such that 0 ≤ f (x) ≤ 1 for all x ∈ M , f −1 (0) = A, and f −1 (1) = B.
By Theorem 2.29, there are functions g, h : M → [0, ∞) such that g −1 (0) = A and h−1 (0) = B. It is
g
immediate to check that g+h has the desired property.

3 Tangent Vectors
Exercise 3.5. Prove Lemma 3.4.
For (a), let f1 : M → R be the constant function at 1. Then

vf1 = v(f12 ) = f1 (p)vf1 + f1 (p)vf1 = 2vf1 ,

so vf1 = 0. If f is the constant function at c ∈ R, then vf = v(cf1 ) = c · vf1 = 0.


For (b), we have
v(f g) = f (p)vg + g(p)vf = 0.
Exercise 3.7. Prove Proposition 3.6.

For (a), let u, v ∈ Tp M be derivations, let a, b ∈ R, and let f ∈ C ∞ (N ). Then

dFp (au + bv)(f ) = (au + bv)(f ◦ F ) = au(f ◦ F ) + bv(f ◦ F ) = adFp (u)(f ) + bdFp (v)(f ).

Since f was arbitrary, we have that dFp (au + bv) = adFp (u) + bdFp (v).
For (b), let v ∈ Tp M be a derivation and let f ∈ C ∞ (P ). Then

d(G ◦ F )p (v)(f ) = v(f ◦ G ◦ F ) = dFp (v)(f ◦ G) = dGF (p) ◦ dFp (v)(f ).

For (c), we have


d(IdM )p (v)(f ) = v(f ◦ IdM ) = vf.
For (d), let F : M → N be a diffeomorphism. Parts (b) and (c) imply IdTp M = d(IdM )p = d(F −1 ◦ F )p =
d(F −1 )F (p) ◦ dFp and IdTp N = d(IdN )F (p) = d(F ◦ F −1 )F (p) = dFp ◦ d(F −1 )F (p) . This shows that dFp is an
isomorphism and that (dFp )−1 = d(F −1 )F (p) .

Exercise 3.17. Let (x, y) denote the standard coordinates on R2 . Verify that (x̃, ỹ) are global smooth
coordinates on R2 , where
x̃ = x, ỹ = y + x3 .

18
Let p be the point (1, 0) ∈ R2 (in standard coordinates), and show that

∂ ∂
6
= ,
∂x p ∂ x̃ p
even though the coordinate functions x and x̃ are identically equal.
The map φ : R2 → R2 , (x, y) 7→ (x, y + x3 ) is smooth with smooth inverse φ−1 : R2 → R2 , (x, y) 7→
(x, y − x3 ). Therefore, (x̃, ỹ) are global smooth coordinates on R2 . Now,

∂ ∂ x̃ ∂ ∂ ỹ ∂ ∂ ∂ ∂
= + = +3 6= .
∂x p ∂x p ∂ x̃ ∂x p ∂ ỹ ∂ x̃ ∂ ỹ ∂ x̃ p
Exercise 3.19. Suppose M is a smooth manifold with boundary. Show that T M has a natural topology
and smooth structure making it into a smooth manifold with boundary, such that if (U, (xi )) is any smooth
boundary chart for M , then rearranging the coordinates in the natural chart (π −1 (U ), (xi , v i )) for T M yields
a boundary chart (π −1 (U ), (v i , xi )).
We begin by defining the topology on T M , which I believe was not done explicitly in Lee. Let A =
{(Uα , φα )} be a smooth atlas for M . For every (U, φ) ∈ A, we have that
φ̃ : π −1 (U ) → φ(U ) × Rn , (p, v i ∂/∂xi p ) 7→ (φ(p), v i ei )

is a bijection. Hence, we obtain a topology on π −1 (U ) by pulling back the topology on φ(U ) × Rn . We then
define a subset of T M to be open if its intersection with each π −1 (U ) is open; it is not hard to check that
this defines a topology on T M . We define the coordinate charts as in the proof of Proposition 3.18, and the
proof that they define a smooth structure on T M is the same.
It remains to show that if (U, φ) is a boundary chart, then (π −1 (U ), φ̃) is a boundary chart (after some
rearranging of coordinates). Simply note that φ(U ) ⊆ Hn , so φ̃(φ−1 (U )) = φ(U ) × Rn ⊆ Hn × Rn ∼ = H2n .
2n n −1
Moreover, φ̃(p, v) ∈ ∂H for any p ∈ U such that φ(p) ∈ ∂H . Hence, (π (U ), φ̃) is a boundary chart. The
statement of the exercise asks us to rearrange coordinates so that the last coordinate of the chart intersects
the boundary, but this is not so important.
Exercise 3.27. Show that any (covariant or contravariant) functor from C to D takes isomorphisms in C
to isomorphisms in D.
Let F be a covariant functor from C to D, and let f ∈ homC (X, Y ) be an isomorphism. Then IdF(X) =
F(f −1 ◦ f ) = F(f −1 ) ◦ F(f ) and IdF (Y ) = F(f ◦ f −1 ) = F(f ) ◦ F(f −1 ). Hence, F(f ) is an isomorphism, with
inverse F(f −1 ). The proof is similar in the case where F is a contravariant functor.
Problem 3.1. Suppose M and N are smooth manifolds with or without boundary, and F : M → N is a
smooth map. Show that dFp : Tp M → TF (p) N is the zero map for each p ∈ M if and only if F is constant
on each component of M .
(⇒) Let dFp be the zero map for each p ∈ M . Let q ∈ N be in the image of F . We will show that F −1 (q)
is both open and closed. Since {q} ⊆ N is closed and F is smooth, F −1 (q) is closed. On the other hand, let
p0 ∈ M be such that F (p0 ) = q, and let (B, φ) be a coordinate ball centred at p0 . If p1 is any other point
in B, then there is a straight-line smooth path γ : [0, 1] → φ(B) such that γ(0) = φ(p0 ) and γ(1) = φ(p1 ).
Then φ−1 ◦ γ is a smooth path from p0 to p1 and F ◦ φ−1 ◦ γ is a smooth path from F (p0 ) to F (p1 ). By
Proposition 3.24, (F ◦ φ−1 ◦ γ)0 (t) = dF ((φ−1 ◦ γ)0 (t)) = 0 for every t ∈ [0, 1].
We will now show that a smooth curve with velocity identically equal to 0 is a constant path. Let P be
a smooth manifold, and let γ : J → N be a path with velocity identically equal to zero (where J ⊆ R is
some interval). Let (V, ψ) be a chart containing γ(t) for some t. Then ψ ◦ γ is a curve in ψ(V ) ⊆ Rn , and
(F ◦ γ)0 (t) = dF (γ 0 (t)) = dF (0) = 0. A smooth curve of velocity 0 in Rn is constant, and therefore γ is
constant.
Hence, the curve F ◦ φ−1 ◦ γ is constant, so F (p0 ) = F (p1 ) = q. We conclude that B ⊆ F −1 (q), so F −1 (q)
is open. Therefore, F −1 (q) is either a component or a union of components. We conclude that F is constant
on each component of M .
(⇐) If F is constant, then since dFp only depends on F in a neighbourhood of p, and F is constant on
the component of p, we have that dFp is the zero map for each p.

19
Problem 3.2. Prove Proposition 3.14.
For each i = 1, . . . , k, let βi : Mi → M1 × · · · × Mk , x 7→ (p1 , . . . , pi−1 , x, pi+1 , . . . , pk ), and let

β : Tp1 M1 ⊕ · · · ⊕ Tpk Mk → Tp (M1 × · · · × Mk ), v1 ⊕ · · · ⊕ vk 7→ d(β1 )p1 (v1 ) + · · · + d(βk )pk (vk ).

It is straightforward to check that β is a linear map. Let α be the map defined in Proposition 3.14. Note
that πi ◦ βi is the identity on Mi if i = j, and is the constant map at pj otherwise. It follows from this
observation and part (b) of Proposition 3.6 that α ◦ β is the identity on Tp1 M1 ⊕ · · · ⊕ Tpk Mk P. Then α is
surjective. Since Tp1 M1 ⊕ · · · ⊕ Tpk Mk and Tp (M1 × · · · × Mk ) have the same dimension (namely i dimMi ),
and α is a surjection, α is an isomorphism.
If one of the Mi is a smooth manifold with boundary, then M1 × · · · × Mk is a smooth manifold with
boundary. Hence it has a well defined tangent space at each point, and differentials of maps are defined.
The same proof then applies in this case.
Problem 3.3. Prove that if M and N are smooth manifolds, then T (M × N ) is diffeomorphic to T M × T N .
By the preceding problem, the tangent space T(p,q) (M × N ) can be identified with Tp M ⊕ Tq N for every
pair (p, q) ∈ M × N . Thus, we can define a map

F : T (M × N ) → T M × T N, ((p, q), u ⊕ v) 7→ ((p, u), (q, v)).

We claim that F is a diffeomorphism. The inverse of F is given by F −1 ((p, u), (q, v)) = ((p, q), u ⊕
v). Let (U, φ) and (V, ψ) be charts for M and N , respectively. Let πM , πN , πM ×N be projections from
−1
T M, T N, T (M × N ) to M, N, M × N , respectively. We have a chart (πM ×N (U × V ), α) for T (M × N ), where
!
−1 m n i ∂ j ∂

α : πM ×N (U × V ) → φ(U ) × ψ(V ) × R × R , (p, q), u ⊕v 7→ (φ(p), ψ(q), u, v).
∂xi (p,q) ∂y j (p,q)

−1 −1
We also have a corresponding chart (πM (U ) × πN (V ), β) for T M × T N , where
! !!
−1 −1 ∂ ∂
(V ) → φ(U ) × Rm × ψ(V ) × Rn , p, ui i , q, v j j

β : πM (U ) × πN 7→ (φ(p), u, ψ(q), v).
∂x p ∂y q

In both maps, u = (u1 , . . . , um ) and v = (v 1 , . . . , v n ). The coordinate representation of F is β ◦ F ◦


α−1 (x, y, u, v) = (x, u, y, v) and the coordinate representation of F −1 is α ◦ F ◦ β −1 (x, u, y, v) = (x, y, u, v).
Both of these maps are smooth, so F is a diffeomorphism.
Problem 3.4. Show that T S1 is diffeomorphic to S1 × R.
View S1 ⊆ C. We have two angle charts (U, θ) and (V, φ) covering S1 , where U = S1 \{1}, θ : U → (0, 2π),
d
and V = S1 \ {−1}, φ : V → (−π, π). Define F : T S1 → S1 × R as follows: if z ∈ U , let F (z, v dθ |z ) = (z, v)
d d d
and if z ∈ V , then let F (z, v dφ |z ) = (z, v). It is not hard to show that dθ |z = dφ |z for z ∈ U ∩ V ; hence,
F is well defined. Note that F restricted to π −1 (U ) or π −1 (V ) is a diffeomorphism from π −1 (U ) to U × R
and from π −1 (V ) to V × R, and it agrees on the overlap π −1 (U ) ∩ π −1 (V ). By the gluing lemma for smooth
maps (Corollary 2.8), F is a diffeomorphism.
Problem 3.5. Let S1 ⊆ R2 be the unit circle, and let K ⊆ R2 be the boundary of the square of side 2 centred
at the origin: K = {(x, y) : max(|x|, |y|) = 1}. Show that there is a homeomorphism F : R2 → R2 such that
F (S1 ) = K, but there is no diffeomorphism with the same property.
Let T1 ⊆ R2 be the set {(x, y) : x ≥ 0, |x| ≥ |y|}. Let r : R2 → R2 be the π/2 angle clockwise rotation
S4
about the origin, and let T2 = r(T1 ), T3 = r2 (T1 ), and T4 = r3 (T1 ). Note that R2 = i=1 Ti . We begin by
defining a homeomorphism F1 : T1 → T1 . For (x, y) ∈ T1 , let
(q 2
1 + xy 2 (x, y) if (x, y) 6= (0, 0)
F1 (x, y) =
0 if (x, y) = (0, 0)

20
q
2 √
For (x, y) 6= (0, 0), we have |F1 (x, y)| = 1 + xy 2 |(x, y)| ≤ 2|(x, y)|, so F1 (x, y) tends to 0 as (x, y) tends
to (0, 0) in T1 . Thus, F1 is continuous. The inverse of F1 is given by
( x
√ 2 2 (x, y) if (x, y) 6= (0, 0)
−1 x +y
F1 (x, y) =
0 if (x, y) = (0, 0)

as one can check. In T1 , we have F1 (x, y) ≤ |(x, y)|, so F1−1 is continuous on T1 . Thus, F1 is a
−1

homeomorphism. We similarly obtain a homeomorphism of Ti via the function Fi = ri−1 ◦ F1 ◦ r−(i−1) .


Finally, define
F : R2 → R2 , (x, y) 7→ Fi (x, y) if (x, y) ∈ Ti .
This function is well defined, since the Fi agree on the overlaps of their domains. By the pasting lemma,
F : R2 → R2 is a homeomorphism. We check that F (S1 ) = K. Assume that (x, y) ∈ S1 ∩ T1 . Then
r
y2 1p 2 1  y
F (x, y) = F1 (x, y) = 1 + 2 (x, y) = x + y 2 (x, y) = (x, y) = 1, ∈ K ∩ T1 .
x x x x
Then F (S1 ) ⊆ K, since S1 and K are both invariant under rotations by π/2. Now let (x, y) ∈ K ∩ T1 . Then
1
F1−1 (x, y) = F1−1 (1, y) = p (1, y),
1 + y2
so F1−1 (x, y) = 1, meaning F1−1 (x, y) ∈ S1 ∩ T1 . Thus, F (S1 ) = K.

Suppose that there is a diffeomorphism F : R2 → R2 such that F (S1 ) = K. Let γ(t) : R → R2 , t 7→


(cos t, sin t). Note that γ(t) ∈ S1 and γ 0 (t) 6= 0 for all t ∈ R. Let tc ∈ R such that F ◦ γ(tc ) = (1, 1). Then,
for some  > 0, there are intervals I− = (tc − , tc ) and I+ = (tc , tc + ) such that F ◦ γ(I− ) ⊆ {1} × (−1, 1)
and F ◦ γ(I+ ) ⊆ (−1, 1) × {1} (assuming that F sends γ counter-clockwise around K). Then
(
(1, y ◦ F ◦ γ(t)) if t ∈ I−
F ◦ γ(t) =
(x ◦ F ◦ γ(t), 1) if t ∈ I+
and
d(x ◦ F ◦ γ) ∂ d(y ◦ F ◦ γ) ∂
(F ◦ γ)0 (tc ) = (tc ) + (t c ) .
dt ∂x (1,1) dt ∂y (1,1)
◦γ) ◦γ)
By continuity of d(x◦F dt and d(y◦F
dt , they must both vanish at tc . Hence, (F ◦γ)0 (tc ) = 0. But Proposition
0 0
3.24 implies (F ◦γ) (tc ) = dF (γ (tc )). Since F is a homeomorphism, dFp is an isomorphism for every p ∈ R2 .
In particular, dF (γ 0 (tc )) = dFγ(tc ) (γ 0 (tc )) 6= 0, since γ 0 (tc ) 6= 0. Thus, F cannot be a diffeomorphism.
Problem 3.6. Consider S3 as the unit sphere in C2 under the usual
 identification C2 ↔ R4 . For each
1 2 3 3 it 1 it 2
z = z , z ∈ S , define a curve γz : R → S by γz (t) = e z , e z . Show that γz is a smooth curve whose
velocity is never zero.
Letting z k = xk + iy k for k = 1, 2, we have that
x1 cos t − y 1 sin t, x1 sin t + y 1 cos t, x2 cos t − y 2 sin t, x2 sin t + y 2 cos t .

γz (t) =
Let (U1+ , φ+ 3
1 ) be the coordinate chart for S described in Example 1.4. The coordinate representation of γz
with respect to this chart is
−1
φ+ + 3 1 1 2 2 2 2

1 ◦ γz : γz (U1 ) → R , t 7→ x sin t + y cos t, x cos t − y sin t, x sin t + y cos t .

Since γz is continuous, γz−1 (U1+ ) is open, and therefore φ+


1 ◦ γz is smooth. A similar computation shows that
the coordinate representation of γz is smooth with respect to every chart (Ui± , φ± i ). The velocity of γz at
some t such that γz (t) ∈ U1+ is
γz0 (t) = (x1 cos t − y 1 sin t, −x2 sin t − y 2 cos t, x2 cos t − y 2 sin t),
in the coordinates of U1+ . Since the first coordinate of γz (t) is x1 cos t − y 1 sin t and is positive for γz (t) in
U1+ , γ 0 (t) has positive first in U1+ and is therefore nonzero. A similar calculation shows that γ 0 (t) is nonzero
in the other charts (Ui± , φ± i ).

21
Problem 3.7. Let M be a smooth manifold with or without boundary and p be a point of M . let Cp∞ (M )
denote the algebra of germs of smooth real-valued functions at p, and let Dp M denote the vector space of
derivations of Cp∞ (M ). Define a map Φ : Dp M → Tp M by (Φv)f = v([f ]p ). Show that Φ is an isomorphism.
Let’s begin by checking that Φv is indeed a derivation at p. It is clearly linear. Let f, g ∈ C ∞ (M ). Then

(Φv)(f g) = v[f g]p = f (p)v[g]p + g(p)v[f ]g = f (p)(Φv)g + g(p)(Φv)f,

so Φv is a derivation at p.
To check that Φ is an isomorphism, we first note that it is clearly linear. For injectivity, let Φv = 0. Let
[f ]p be the germ of some pair (f, U ), where U is open an contains p. Let B 0 be a coordinate ball contained
in U and centred at p, and let B be a coordinate ball centred at p such that B ⊆ B. Moreover, by the
extension lemma for smooth functions, there is a smooth function ψ supported in U such that ψ ≡ 1 on U .
Then f˜ = ψf is a smooth function such that [f˜]p = [f ]p . Then v[f ]p = v[f˜]p = (Φv)(f˜) = 0. Since [f ]p was
arbitrary, we conclude that v = 0, and therefore that Φ is injective.
Finally, let w ∈ Tp M be an arbitrary derivation. Define v ∈ Dp M by v[f ]p = wf . Since wf = wg if
f = g on some neighbourhood of p (Proposition 3.8), v is well defined. Then (Φv)f = v[f ]p = wf for any
f ∈ C ∞ M , so Φv = w. This proves surjectivity, and therefore proves that Ψ is an isomorphism.
Problem 3.8. Let M be a smooth manifold with or without boundary and p ∈ M . Let Vp M denote the set
of equivalence classes of smooth curves starting at p under the relation γ1 ∼ γ2 if (f ◦ γ1 )0 (0) = (f ◦ γ2 )0 (0)
for every smooth real-valued function defined on a neighbourhood of p. Show that the map Ψ : Vp M → Tp M
defined by Ψ[γ] = γ 0 (0) is well defined and bijective.
Let [γ1 ] = [γ2 ] for two smooth curves starting at p. Let (U, φ) be a chart containing p. In the coordinates
of this chart, we have

∂ ∂
γ10 (0) = (xi ◦ γ1 )0 (0) = (x i
◦ γ 2 ) 0
(0) = γ20 (0),
∂xi p ∂xi p

since (f ◦ γ1 )0 (0) = (f ◦ γ2 )0 (0) for every smooth real-valued function f defined in a neighbourhood of p.
Hence, Ψ is well defined.
For injectivity, let γ10 (0) = γ20 (0), and let f be any smooth real-valued function defined on a neighbourhood
of p. Then
   
d d
(f ◦ γ1 )0 (0) = d (f ◦ γ)0 = df γ(0) ◦ dγ0 = dfγ(0) ◦ γ10 (0) = dfγ(0) ◦ γ20 (0) = (f ◦ γ2 )0 (0),
dt 0
dt 0

so [γ1 ] = [γ2 ], proving injectivity.


For surjectivity, let v be an arbitrary derivation at p, and let (U, φ) be some coordinate chart containing
p such that φ(p) = 0. Then we can write
i ∂

v = v i
∂x p

in the coordinates of (U, φ). For some  > 0, we can define a curve

γ̃ : [0, ) → U, t 7→ (tv 1 , . . . , tv n ),

and then let γ = φ−1 ◦ γ̃. Then γ is a smooth curve starting at p, since it is the composition of smooth
functions. Then
dγ i d(φi ◦ γ) d(tv 1 )

0 ∂ ∂ ∂ i ∂

γ (0) = (0) = (0) = (0) = v = v.
dt ∂xi γ(0) dt ∂xi γ(0) dt ∂xi γ(0) ∂xi p

4 Submersions, Immersions, and Embeddings


Exercise 4.3. Verify the claims made in the preceding example.

22
For (a), let (U1 × · · · × Uk , φ1 × · · · × φk ) be a smooth chart for M1 × · · · × Mk . Then πi has the coordinate
representation
φi ◦ πi ◦ (φ1 × · · · × φk )−1 (v1 , . . . , vk ) = vi .
Let dim Mi = ni and let vi = (x1i , . . . , xni 1 ). Thus, in coordinates, d(πi )p is the (n1 + · · · + nk ) × ni matrix
(∂xji /∂xlm ), where j ranges from 1 to ni , m ranges from 1 to k, and for each m, l ranges from 1 to nm . This
matrix is of full rank, since the only nonzero terms occur when j = l and i = m simultaneously. Hence,
d(πi )p is surjective and is therefore a smooth submersion.
For (b), suppose that the smooth curve γ is a smooth immersion. Then dγt0 is injective for every t0 ∈ J.
Then γ 0 (t0 ) = dγt0 ( d/dt|t0 ) 6= 0. Conversely, suppose γ 0 (t0 ) 6= 0 for every t0 ∈ J. Suppose that dγt0 (v) = 0
for some v ∈ Tt0 J. Since Tt0 J is spanned by d/dt|t0 , we have that v = α d/dt|t0 for some α ∈ R. Then
0 = dγt0 (α d/dt|t0 ) = αdγt0 ( d/dt|t0 ) = αγ 0 (t0 ), implying α = 0. Hence, dγt0 is injective, and therefore γ is
a smooth immersion.
For (c), let (U, φ) be an open chart on M , and let (π −1 (U ), φ̃) be the corresponding chart on on T M .
Then the coordinate representation of π with respect to these charts is φ ◦ π ◦ φ̃−1 (x, v) = x. A similar
computation as the one done in (a) shows that dπ|p is a surjection for every p ∈ T M , and therefore π is a
submersion.
For (d), dX(u,v) has the matrix representation
 
−2π sin 2πu cos 2πv −2π(2 + cos 2πu) sin 2πv
dX(u,v) =  −2π sin 2πu sin 2πv 2π(2 + cos 2πu) cos 2πv  .
2π cos 2πu 0

Note that at least one of the (1, 2) or (2, 2) entries is nonzero. If 2π cos 2πu = 0, then the bottom row is
a row of zeros, and the determinant of the 2 × 2 matrix formed by the top two rows is ±8π. Hence, the
columns are linearly independent, and thus dX(u,v) has full rank. Therefore, X is a smooth immersion.
Exercise 4.4. Show that a composition of smooth submersions is a smooth submersion, and a composition
of smooth immersion is a smooth immersion. Give a counterexample to show that a composition of maps of
constant rank need not have constant rank.
Let F : M → N and G : N → P be smooth [submersions/immersions]. Then for every p ∈ M , dFp and
dGF (p) are [surjective/injective]. Hence, dGF (p) ◦ dFp = d(G ◦ F )p is [surjective/injective]. Therefore, G ◦ F
is a smooth [submersion/immersion].
For the counterexample, let f : R → R2 , x 7→ (x, x2 ) and g : R2 → R, (x, y) 7→ y. Note that g is a a
submersion and therefore has constant rank 1. Moreover, dfx = (1 2x)> , so f has constant rank 1. However,
g ◦ f : R → R, x 7→ x2 has derivative d(g ◦ f )x = 2x, so g ◦ f has rank 1 for each x 6= 0 and rank 0 at x = 0.
Exercise 4.7. Prove Proposition 4.6.
For (a), let F : M → N and G : N → P be local diffeomorphisms and let p ∈ M . Let U ⊆ M be an open
set containing p such that F |U : U → F (U ) is a diffeomorphism. Let V ⊆ N be an open set containing F (p)
such that G|V : V → G(V ) is a a diffeomorphism. Then G◦F |U ∩F −1 (V ) : U ∩F −1 (V ) → (G◦F )(U )∩G(V ) is
a diffeomorphism, where U ∩F −1 (V ) ⊆ M is an open set containing p. Hence, G◦F is a local diffeomorphism.
For (b), let Fi : Mi → Ni be a local homeomorphism for i = 1, . . . , k, and consider the product

F1 × · · · × Fk : M1 × · · · × Mk → N1 × · · · × Nk .

Let p = (p1 , . . . , pk ) ∈ M1 ×· · ·×Mk and let Ui be open an open set containing pi such that Fi |Ui : Ui → Fi (Ui )
is a diffeomorphism. By Exercise 2.16,

F1 |U1 × · · · × Fk |Uk : U1 × · · · × Uk → F1 (U1 ) × · · · × Fk (Uk )

is a diffeomorphism. Since this is the same map as

(F1 × · · · × Fk )|U1 ×···×Uk : U1 × · · · × Uk → (F1 × · · · × Fk )(U1 × · · · × Uk ),

we have that F1 × · · · × Fk is a local diffeomorphism.

23
For (c), let F : M → N be a local diffeomorphism. Since every point p ∈ M has an open neighbourhood
U such that F |U : U → F (U ) is a diffeomorphism. Since diffeomorphisms are homeomorphisms, F is a
local homeomorphism. Let U ∈ M be an arbitrary open set. Let p0 ∈ F (U ) and let p ∈ U be such that
F (p) = p0 . Since U is open, we can choose an open set V such that p ∈ V ⊆ U and F |V : V → F (V ) is a
diffeomorphism. Then F (V ) ⊆ F (U ) is open and contains p0 . Thus, F (U ) is open and F is an open map.
For (d), let F : M → N be a local diffeomorphism and let U ⊆ M be an open submanifold. Let p ∈ U
be arbitrary, and let V ⊆ M be an open set such that F |V : V → F (V ) is a diffeomorphism. By Exercise
2.16, (F |V )|U ∩V = F |(U ∩V ) : U ∩ V → F (U ∩ V ) is a diffeomorphism. Hence, F |U is a local diffeomorphism.
For (e), let F : M → N be a diffeomorphism and let p ∈ M . Note that M itself is an open set containing
p and F |M = F is a diffeomorphism. Therefore, F is a local diffeomorphism.
For (f), let F : M → N be a bijective local diffeomorphism. To see that F is smooth, note that every point
of M is contained in a chart (U, φ) such that F |U is a diffeomorphism. Then the coordinate representation
of F in the chart (U, φ) is smooth, so F is smooth. To see that F −1 is also smooth, let F (p) ∈ N and let
U ⊆ M be an open set containing p such that F |U is a diffeomorphism. Then F −1 |F (U ) : F (U ) → U is
also a diffeomorphism. Let (V, ψ) be a smooth chart containing F (p) such that V ⊆ F (U ). Then F −1 |V
is a diffeomorphism onto its image, and therefore F −1 has a smooth coordinate representation in (V, ψ).
Therefore, F −1 is smooth and F is a diffeomorphism.
For (g), let F : M → N be a local diffeomorphism. Let p ∈ M and let U be an open set containing p
such that F |U : U → F (U ) is a diffeomorphism. We can then find a chart (V, φ) such that V ⊆ U . Then the
coordinate representation of F in (V, φ) is a diffeomorphism, and therefore a local diffeomorphism by part
(e). Conversely, suppose that every point p ∈ M is contained in a chart (U, φ) such that F (p) is in some chart
(V, ψ) and ψ ◦ F ◦ φ−1 : φ(U ∩ F −1 (V )) → ψ(F (U ∩ F −1 (V ))) is a local diffeomorphism. Since F is smooth
(coordinate representations are only defined for smooth maps), φ(U ∩ F −1 (V )) is open in Rn (or Hn ). Since
local diffeomorphisms are open maps, ψ(F (U ∩F −1 (V ))) is open in Rn (or Hn ). Since the restriction of a local
diffeomorphism to an open submanifold is a local diffeomorphism, (ψ◦F ◦φ−1 )◦φ|U ∩F −1 (V ) = ψ◦F |U ∩F −1 (V ) .
Similarly, ψ −1 |ψ(F (U ∩F −1 (V ))) is a local diffeomorphism, and therefore F |U ∩F −1 (V ) : U ∩ F −1 (V ) → F (U ∩
F −1 (V )) is a local diffeomorphism. Then there is an open set W ⊆ U ∩ F −1 (V ) containing p such that
F |W : W → F (W ) is a diffeomorphism. Hence, F is a local diffeomorphism.

Exercise 4.9. Show that the conclusions of Proposition 4.8 still hold if N is allowed to be a smooth manifold
with boundary, but not if M is.
To show that the result holds when N has nonempty boundary, we just need to show that we can apply
the inverse function theorem for manifolds when F is a smooth immersion and a smooth submersion. To do
this, it suffices to show that F (M ) ⊆ IntN . This is the result of 4.2.
The inclusion map Hn ,→ Rn is both a smooth immersion and a smooth submersion (it’s differential
is given by the identity). On the other hand, it is not a local diffeomorphism: if U ⊆ Hn is an open
neighbourhood of 0, then F (U ) is not open in Rn . In particular, F (U ) contains no open ball centred at 0.
Exercise 4.10. Suppose M, N, P are smooth manifolds with or without boundary, and F : M → N is a
local diffeomorphism. Prove the following:
(a) If G : P → M is continuous, then G is smooth if and only if F ◦ G is smooth.
(b) If in addition F is surjective and G : N → P is any map, then G is smooth if and only if G ◦ F is
smooth.
For (a), let G be smooth. Since local diffeomorphisms are smooth, F ◦ G is smooth. Conversely, suppose
that F ◦ G is smooth. Let p ∈ P , and let U ⊆ M be an open set containing g(p) such that F |U : U → F (U )
is a diffeomorphism. Since G is continuous, G−1 (U ) is open, and therefore (F ◦ G)|G−1 (U ) : G−1 (U ) → F (U )
is smooth. Then (F |U )−1 ◦ (F ◦ G)|G−1 (U ) = G|G−1 (U ) : G−1 (U ) → U is smooth. Thus, G is smooth.
For (b), let G be smooth. Since local diffeomorphisms are smooth, G ◦ F is smooth. Conversely, suppose
that G ◦ F is smooth. Let q ∈ N be arbitrary, and let F (p) = q (using the fact that F is surjective). Let
U ⊆ M be an open neighbourhood op p such that F |U : U → F (U ) is a diffeomorphism. Then G ◦ F |U is
smooth, and so is (G ◦ F |U ) ◦ (F |U )−1 = G|F (U ) : F (U ) → N . Thus, G is smooth.
Exercise 4.17. Show that every composition of smooth embeddings is a smooth embedding.

24
The composition of smooth immersions is a smooth immersion (Exercise 4.4). It remains to show that
a composition of topological embeddings is a topological embedding. Let F : M → N and G : N → P
be topological embeddings. Then F : M → F (M ) and G : N → G(N ) are homeomorphisms. Then
G|F (M ) : F (M ) → G ◦ F (M ) is a homeomorphism, so G ◦ F : M → G ◦ F (M ) is a homeomorphism.
Therefore G ◦ F is a topological embedding.
Exercise 4.24. Give an example of a smooth embedding that is neither an open map nor a closed map.
Let F : R → R2 be given by F (t) = (et , 0). Then F is a continuous bijection onto (0, ∞) × {0} and
−1
F (x, 0) = log x for x > 0. The inverse is continuous, so we have that F is a topological embedding. Next,
dFt = (et 0) is of full rank for all t, so F is a smooth immersion and therefore is a smooth embedding. Note
that F (R) = (0, ∞) × {0} is neither open nor closed in R2 despite the fact that R is both open and closed
in R. Hence, F is neither an open nor a closed map.
Exercise 4.27. Give an example of a smooth map that is a topological submersion but not a smooth sub-
mersion.
Let F : R → R be given by F (x) = x3 . Then F is smooth and bijective with continuous inverse
−1
F (x) = x1/3 . Therefore F is a topological submersion. However, dF0 = 0, so F is not a smooth
submersion.
Exercise 4.32. Prove Theorem 4.31.
By Theorem 4.30, there are smooth maps F : N1 → N2 and G : N2 → N1 such that F ◦ π2 = π1 and
G ◦ π1 = π2 . Then G ◦ F ◦ π2 = π2 . Since π2 is surjective, G ◦ F = Id. A similar argument shows that
F ◦ G = Id. Hence, G = F −1 and F is a diffeomorphism.
M
π2 π1

N1 N2
G

Exercise 4.34. Prove Proposition 4.33.


For (a), let π : E → M be a smooth covering map. Let p ∈ M and let q ∈ E such that π(q) = p. There
is a neighbourhood U of p such that V ⊆ E is the component of π −1 (U ) containing q. Then π|V : V → U is
a diffeomorphism, meaning π is a local diffeomorphism. By Proposition 4.8, π is also a smooth submersion
(the fact that E and M can have nonempty boundary is not a problem for this direction of the proof of
Proposition 4.8). Since local diffeomorphisms are open maps, π is an open map. Surjective open maps are
quotient maps, so π is a quotient map.
For (b), if π is injective, then it is bijective. Since bijective local diffeomorphisms are diffeomorphisms, π
is a diffeomorphism.
For (c), let π : E → M be a topological covering map that is also a smooth covering map. By part (a), π
is also a local diffeomorphism. Conversely, let π : E → M be a topological covering map that is also S a local
diffeomorphism. Let p ∈ M and let U be an evenly covered neighbourhood of p, and let π −1 (U ) = α∈A Uα ,
where π|Uα is a homeomorphism onto its image. Then π|Uα is a bijective local diffeomorphism, and therefore
a diffeomorphism.
Exercise 4.37. Suppose π : E → M is a smooth covering map. Show that every local section of π is smooth.
Let U ⊆ M be open, and let σ : U → E be a local section. Note that every p ∈ U is contained in an
evenly covered open neighbourhood Up ⊆ U . By Proposition 4.36, there is a unique smooth local section
τ : Up → E of π such that τ (p) = σ(p). Then π ◦ σ|Up = π ◦ τ , and since π is bijective on π −1 (Up ), we have
that σ|Up = τ . Therefore, σ is smooth.
Exercise 4.38. Suppose E1 , . . . , Ek and M1 , . . . , Mk are smooth manifolds (without boundary), and πi :
Ei → Mi is a smooth covering map for each i = 1, . . . , k. Show that π1 ×· · ·×πk : E1 ×· · ·×Ek → M1 ×· · ·×Mk
is a smooth covering map.

25
We already have that π1 × · · · × πk is smooth and surjective. Let pi ∈ Mi be an arbitrary point, and let
Ui be an evenly covered neighbourhood of pi for each i = 1, . . . , k. Let Ui,αi be a component of π −1 (Ui ) for
each i = 1, . . . , k. Since the product of diffeomorphisms is a diffeomorphism, we have that

(π1 )|U1,α1 × · · · × (πk )|Uk,αk : U1,α1 × · · · × Uk,αk → U1 × · · · × Uk

is a diffeomorphism. Since the connected components of (π1 × · · · × πk )−1 (U1 × · · · × Uk ) are all of the form
U1,α1 × · · · × Uk,αk , this shows that π1 × · · · × πk is a smooth covering map.
Exercise 4.39. Suppose that π : E → M is a smooth covering map. Since π is also a topological covering
map, there is a potential ambiguity about what it means for a subset U ⊆ M to be evenly covered: does π
map the components of π −1 (U ) diffeomorphically onto U , or merely homeomorphically? Show that the two
concepts are equivalent: if U ⊆ M is evenly covered in the topological sense, then π maps each component of
π −1 (U ) diffeomorphically onto U .
Let V be a component of π −1 (U ). Note that every p ∈ U has an open neighbourhood Up ⊆ U such that
π|V ∩π−1 (Up ) : V ∩ π −1 (Up ) → Up is a diffeomorphism. Hence, π|V is a bijective local diffeomorphism, and
therefore a diffeomorphism. Hence, π maps the components of π −1 (U ) diffeomorphically onto U .
Exercise 4.42. Prove Proposition 4.41.
The facts that E is second-countable and Hausdorff are proven in exactly the same way as in the proof
of Proposition 4.40. To show that E is a topological n-manifold with boundary, let p ∈ E and first suppose
that π(p) ∈ ∂M . Then there is an evenly covered open neighbourhood U of π(p) such that (U, φ) is also
a boundary chart for M . Letting V be the component of π −1 (U ) containing p, we find that (V, φ ◦ π|V ) is
a boundary chart containing p. We can similarly construct an interior boundary chart of any point p ∈ E
mapping to IntM . Therefore, E is a topological n-manifold with boundary.
Let p ∈ ∂E and let U ⊆ M be an evenly covered open neighbourhood of π(p), such that (U, φ) is a chart
with domain U . As described above, we obtain a chart (V, φ ◦ π|V ) containing p. Since p is a boundary
point, φ(π(p)) ∈ ∂Hn , which implies that π(p) is a boundary point of M . Then p ∈ π −1 (π(p)) ⊆ π −1 (∂M ).
Conversely, suppose that p ∈ π −1 (∂M ). Let U be an evenly covered open neighbourhood of π(p) that is also
the domain of a chart (U, φ). Then φ(π(p)) ∈ ∂Hn , which implies that p is a boundary point of the chart
(V, φ ◦ π|V ), and therefore of E. Thus, we have proven ∂E = π −1 (∂M ).
To prove the existence of a unique smooth structure on E making π into a smooth covering map, we
need to show that the charts (V, φ ◦ π|V ) constructed above are smoothly compatible. This is done in the
same way as in the proof of Proposition 4.40. The proof of uniqueness will be given in Problem 4.9.
Exercise 4.44. Prove Corollary 4.43.
We know that any connected and locally simply connected space admits a unique universal cover. Since
topological manifolds admit bases of coordinate balls, they are locally simply connected. Thus, if M is
a connected (smooth) manifold, it admits a topological covering map π : M f → M where M̃ is simply
connected. By Proposition 4.40, M has a unique smooth structure making π into a smooth covering map.
f
This proves the existence of a smooth covering manifold of M .
For uniqueness, suppose that πb:Mc → M is another smooth covering of M by a simply connected space
M . By uniqueness of topological universal coverings, there is a homeomorphism φ making the following
f
diagram commute
φ
M
f M
c
.
π π
b
M

Every point of M has a neighbourhood that is evenly covered by π and π b (by intersecting two possibly
different evenly covered sets). Let V ⊆ M be a component of π −1 (U ). Then φ(V ) is a component of π
b−1 (U ),
and we have the following commutative diagram

26
U
(π|V )−1 π |φ(V ) )−1
(b
.
φ|V
V φ(V )

By Theorem 4.29, since (π|V )−1 is a surjective smooth submersion (it is a diffeomorphism) φ|V ◦ (π|V )−1
is smooth if and only if φ|V is smooth. Since φ|V ◦ (π|V )−1 = (b
π |φ(V ) )−1 is smooth, we have that φ|V is
smooth. Since M is covered by sets of the form V , φ is smooth. A similar argument shows that φ−1 is
smooth, so φ is a diffeomorphism, which proves uniqueness of the universal covering manifold of M .
Exercise 4.45. Generalize the preceding corollary to smooth manifolds with boundary.

By Proposition 4.41, every smooth manifold with boundary M admits a universal covering π : Mf→M
where M is a smooth manifold with boundary.
f
We will prove uniqueness in a simpler way than we did in Exercise 4.44 that does not require the use
of Theorem 4.29 (which is not stated for manifolds with boundary). Again, if there is another universal
covering π c → M , then there is a homeomorphism φ making
b:M
φ
M
f M
c
.
π π
b
M

commute. Let U ⊆ M be an evenly covered neighbourhood, and let V be a component of π −1 (U ). Then φ(V )
b−1 (U ), and we have that π
is a component of π b|φ(V ) ◦ φ|V = π|V . Since π
b|φ(V ) and π|V are diffeomorphisms,
we have that φ|V is a diffeomorphism. Then φ is a bijective local diffeomorphism, and therefore is a
diffeomorphism. This concludes the proof of uniqueness.

Problem 4.1. Use the inclusion map Hn ,→ Rn to show that Theorem 4.5 does not extend to the case in
which M is a manifold with boundary.
Denote the inclusion by i : Hn ,→ Rn . Then i is smooth and di(0,0) = Id is invertible. Suppose there are
connected open neighbourhoods U0 ⊆ Hn of 0 and V0 ⊆ Rn of 0 such that i|U0 : U0 → V0 is a diffeomorphism.
But U0 is a manifold with nonempty boundary, while V0 has empty boundary. This shows that Theorem 4.5
(the inverse function theorem for manifolds) does not hold when the domain is a manifold with boundary.
Problem 4.2. Suppose M is a smooth manifold (without boundary), N is a smooth manifold with boundary,
and F : M → N is smooth. Show that if p ∈ M is a point such that dFp is nonsingular, then F (p) ∈ IntN .
Suppose there is a p ∈ M such that F (p) ∈ ∂N . Let (U, φ) be a chart containing p and let (V, ψ) be a
boundary chart containing F (p). Suppose that the coordinates of U are xi for i = 1, . . . , m and those of V are
y i (x1 , . . . , xm ) for i = 1, . . . , n. Then y n (x1 , . . . , xn ) has a local minimum at (x1 , . . . , xm ) = φ(p). Therefore,
∂y n /∂xi |φ(p) = 0 for each i = 1, . . . , m, which implies that the bottom row of the matrix representation of
dFp is a row of zeros. Hence, dFp is singular, a contradiction.
Problem 4.3. Formulate and prove a version of the rank theorem for a map of constant rank whose domain
is a smooth manifold with boundary.

Formulation of the “domain manifold with boundary” version of the rank theorem: let M be a smooth
manifold with boundary and let N be a smooth manifold of dimensions m and n, respectively, and let
F : M → N be a smooth map with constant rank r. For each p ∈ M there exist smooth charts (U, φ)
for M centred at p and (V, ψ) for N centred at F (p) such that F (U ) ⊆ V , in which F has a coordinate
representation of the form

Fb(x1 , . . . , xr , xr+1 , . . . , xm ) = (x1 , . . . , xr , 0, . . . , 0).

We now begin the proof of this version of the rank theorem. After choosing smooth coordinates, we can
replace M and N by open subsets U ⊆ Hm and V ⊆ Rn . Moreover, we may assume that U intersects ∂Hm ,

27
otherwise the proof is the same as the one given in the text. By translating, we may assume that p = 0 and
F (p) = 0. Since F is smooth, it has an extension Fe : U e → V , where U ⊆ U e and U e is open in Rm . We will
n r
now show that there is a projection π : R → R such that π ◦ Fe is a submersion. Since F has constant rank
r, the matrix (∂ Fei /∂xj ) has an r × r submatrix with nonzero determinant at (x1 , . . . , xm ) = 0 (assume it is
the upper-left submatrix by reordering coordinates). Therefore, by shrinking U e if necessary, we may assume
that this submatrix has non-vanishing determinant on all of U . Let π : R → Rr be the projection onto the
e n

first r coordinates; then π ◦ Fe is a submersion.


By the usual rank theorem, for each q ∈ U e , there are charts (U0 , φ0 ) for U
e centred at q and (W, α) for
π(V ) centred at π ◦ Fe(q) such that π ◦ Fe(U0 ) ⊆ W and

α ◦ π ◦ Fe ◦ φ−1 1 r
0 (x , . . . , x , x
r+1
, . . . , xm ) = (x1 , . . . , xr ).

Note that α ◦ π ◦ Fe ◦ φ−1 −1


0 = π ◦ (α × idRn−r ) ◦ F ◦ φ0 , so
e

(α × idRn−r ) ◦ Fe ◦ φ−1 1 r
0 (x , . . . , x , x
r+1
, . . . , xm ) = (x1 , . . . , xr , R(x1 , . . . , xr , xr+1 , . . . , xm ))

for some smooth function R defined on φ0 (U0 ). By shrinking U0 if necessary, we may assume that φ0 (U0 ) is
an open cube. By the same argument given in the proof of the usual rank theorem, R|φ0 (U0 ∩U ) is independent
of xr+1 , . . . , xm (this uses that F has constant rank r).
Define the open set V0 = {(y 1 , . . . , y r , y r+1 , . . . , y n ) ⊆ (W × Rn−r ) ∩ V : (y 1 , . . . , y r , 0, . . . , 0) ⊆ φ0 (U0 )}.
Moreover, define ψ0 : V0 → Rn by ψ0 (v, w) = (v, w −R(v, 0)), where v = (y 1 , . . . , y r ) and w = (y r+1 , . . . , y n ).
Note that ψ0 is a diffeomorphism onto its image since it has inverse given by ψ0−1 (v, w) = (v, w + R(v, 0)).
Therefore, (V0 , ψ0 ) is a chart for V centred at 0. Note that on φ0 (U0 ∩ U ) we have

ψ0 ◦ (α × idRn−r ) ◦ F ◦ φ−1 1 r
0 (x , . . . , x , x
r+1
, . . . , xm ) = (x1 , . . . , xr , 0, . . . , 0)

since R is independent of xr+1 , . . . , xm there.


What remains is to show how (U0 , φ0 ) can be made into a boundary chart for U . Simple restriction will
not work here and an idea similar to the one used in the proof of Theorem 4.15 will not work, since we do
not have that F is an immersion. Unfortunately, I don’t know how to do this step.
Problem 4.4. Let γ : R → T2 be the curve of Example 4.20. Show that the image set γ(R) is dense in T2 .
We claim that for every t ∈ R, the set {e2πiα(t+n) : n ∈ Z} is dense in S1 . Since multiplication by e2πiαt
defines a homeomorphism of S1 , it is enough to show that {e2πiαn : n ∈ Z} is dense in S1 . It is then enough
k k+1
to show that for every N ∈ N and every k = 0, . . . , N − 1, there is an n ∈ Z such that bαnc ∈ ( N , N ),
k k+1
or equivalently, that there exists an m ∈ Z such that αn − m ∈ ( N , N ). By Dirichlet’s approximation
theorem, there are m0 , n0 ∈ Z such that |αn0 − m0 | < N1 . If αn0 − m0 ∈ (− N1 , 0), replace m0 and n0 by −m0
and −n0 , respectively, so that αn0 − m0 ∈ (0, N1 ). Note that we are using the fact that α is irrational here to
justify our use of open intervals. Then, for some l ∈ N, we have that l(αn0 − m0 ) = α(ln0 ) − (lm0 ) ∈ ( N
k k+1
, N ).
0 0
Choose m = lm and n = ln .
Now let U ⊆ S1 × S 1 be open. We may then assume that U is of the form U1 × U2 , where Ui ⊆ S1 is
open. Choose t ∈ R such that e2πit ∈ U . By our previous work, there is an n ∈ Z such that e2πiα(t+n) ∈ U2 .
Hence,
γ(t + n) = (e2πi(t+n) , e2πiα(t+n) ) = (e2πit , e2πiα(t+n) ) ∈ U1 × U2 .
Hence, γ(R) intersects every open set in T2 , implying that γ(R) is dense in T2 .
Problem 4.5. Let CPn denote the n-dimensional complex projective space, as define in 1.9.
(a) Show that the quotient map π : Cn+1 \ {0} → CPn is a surjective smooth submersion.
(b) Show that CP1 is diffeomorphic to S2 .
ek = {(z 1 , . . . , z n+1 ) ∈ Cn+1 \ {0} : z k 6= 0} and let Uk = π(U
For (a), let U ek ). Let (z 1 , . . . , z n+1 ) ∈ U
ek .
Then, with respect to the charts (U ek , id) and (Uk , φk ) (see Problem 1.9), the coordinate representation of π
is  1
z k−1 z k+1 z n+1

z
φk ◦ π(z 1 , . . . , z n+1 ) = , . . . , , , . . . , ,
zk zk zk zk

28
which is smooth since z k 6= 0. Hence, π is smooth and surjective. It remains to show that π is a submersion.
Letting z j = xj + iy j , we have that

zj xj xk + y j y k xk y j − xj y k
= + i .
zk (xk )2 + (y k )2 (xk )2 + (y k )2

In the coordinates of Ui , one can then check that dπ has a 2n × 2n submatrix of the form
 
D 0 ··· 0
0 D ··· 0
A = . ..  ,
 
.. . .
 .. . . .
0 0 ··· D

where
xk −y k
!
(xk )2 +(y k )2 (xk )2 +(y k )2
D = yk xk
.
(xk )2 +(y k )2 (xk )2 +(y k )2

Then det A = 1, so dπ is of full rank, and therefore π is a submersion.


For (b), we define a diffeomorphism F : S2 → CP1 as follows:
(
x y
[1, 1−z + i 1−z ] if (x, y, z) ∈ S2 \ {N }
F (x, y, z) = x y
[ 1+z − i 1+z , 1] if (x, y, z) ∈ S2 \ {S}.

We check that F is well defined: if (x, y, z) ∈ S2 \ {N, S}, then


      
x y x y x y x y
1, +i = −i 1, +i = −i ,1 .
1−z 1−z 1+z 1+z 1−z 1−z 1+z 1+z

Next, note that F |S2 \{N } = φ−1 1


2 ◦ i ◦ σ, where φ2 is one of the standard coordinate maps defined on CP , i is
2
the identification of R with C, and σ is the stereographic projection from the north. Since each of these are
diffeomorphisms (i is by definition essentially), we have that F |S2 \{N } is a diffeomorphism onto its image.
Similarly, F |S2 \{S} = φ−1
1 ◦ τ ◦ i ◦ σ̃, where τ is complex conjugation and σ̃ is the stereographic projection
from the south. Thus, F |S2 \{S} is also a diffeomorphism onto its image. Since both restrictions are defined
on open sets and they agree on the overlap, we have that F is a diffeomorphism onto its image. It remains to
show that F is surjective. This is straightforward, since the image of F |S2 \{N } is U1 , the image of F |S2 \{S}
is U2 , and CP1 = U1 ∪ U2 . Hence, CP1 is diffeomorphic to S1 .
Problem 4.6. Let M be a nonempty smooth compact manifold. Show that there is no smooth submersion
F : M → Rk for any k > 0.
By Proposition 4.28, if F is a smooth submersion then it is an open map. Since M is open in itself,
F (M ) is open. Since M is compact and F is continuous, F (M ) is compact. Since open sets in Rk are not
compact, this is a contradiction. Hence, there is no smooth submersion from M to Rk . Note that this result
can easily be generalized: if M is a compact topological space, then there is no open map F : M → Rk for
k > 0.
Problem 4.7. Suppose M and N are smooth manifolds, and π : M → N is a surjective submersion. Show
that there is no other smooth manifold structure on N that satisfies the conclusion of Theorem 4.29; in other
words, assuming that N e represents the same set as N with a possibly different topology and smooth structure,
and that for every smooth manifold P with or without boundary, a map F : N e → P is smooth if and only if
F ◦ π is smooth, show that IdN is a diffeomorphism between N and N . e

Let π̃ : M → N e be the same smooth surjective submersion as π, but viewed as a map into N e . Note
−1 −1
that π : M → N is equal to IdN ◦ π e and is smooth. Therefore IdN is smooth. Note that IdNe : N → N
e e is
smooth, and therefore that IdNe ◦ π̃ = π̃ is smooth. Then IdN is smooth, since IdN ◦ π = π̃ is smooth. This
shows that IdN is a diffeomorphism.

29
Problem 4.8. This problem shows that the converse of Theorem 4.29 is false. Let π : R2 → R be defined
by π(x, y) = xy. Show that π is surjective and smooth, and for each smooth manifold P , a map F : R → P
is smooth if and only if F ◦ π is smooth; but π is not a smooth submersion.
Since π(x, 1) = x for all x ∈ R, we have that π is surjective. Moreover, π is smooth since it is a polynomial.
Suppose that F : R → P is smooth; then F ◦ π is smooth by composition. Conversely, suppose that F ◦ π is
smooth. Since f : R → R2 , x 7→ (x, 1) is smooth, we have that F ◦ π ◦ f = F is smooth by composition. To
see that π is not a submersion, note that
 
y
dπ(x,y) = ,
x

which has rank 0 at (0, 0).


Problem 4.9. Let M be a connected smooth manifold, and π : E → M be a topological covering map.
Complete the proof of Proposition 4.40 by showing that there is only one smooth structure on E such that π
is a smooth covering map.
We will prove this for manifolds with boundary as well. We already know that E has a smooth structure
making π : E → M a smooth covering map. Suppose that E e is the same topological manifold as E, but
with a possibly different smooth structure making π̃ : E → M into a smooth covering map, where π̃ is the
e
same map as π, just viewed as a map whose domain is E. e We will prove that the identity map Id : E → E e
is a diffeomorphism; it suffices to show that Id is a local diffeomorphism since it is obviously bijective.
Every point p ∈ E is in the preimage of some evenly covered V ⊆ M . Let U be the component of π −1 (U )
containing p. Since there is no distinction between being smoothly evenly covered and topologically evenly
covered (Exercise 4.39), Id(U ) is also a component of π̃ −1 (V ), and there is a smooth section σ : V → Id(U )
of π̃. Thus, Id|U = σ ◦ π|U is a diffeomorphism, so Id is a local diffeomorphism, proving our claim.
Problem 4.10. Show that the map q : Sn → RPn defined in Example 2.13(f ) is a smooth covering map.
As discussed in Example 2.13(f), we have that q is smooth. By our solution to Exercise 1.7, we have that
q is surjective as well. Consider the usual smooth charts (Ui , φi ) for RPn and the smooth charts (Vi± , ψi± )
for Sn (see Example 1.4) for i = 1, . . . , n + 1. It is not too hard to see that q −1 (Ui ) = Vi+ t Vi− . Hence, if
we can show that q|V ± : Vi± → Ui is a diffeomorphism, we will be done. In coordinates, we have
i

(x1 , . . . , xn )
φi ◦ q|V ± ◦ (ψi± )−1 (x1 , . . . , xn ) = ± q .
i 2
1 − |x|

Then φi ◦ q|V ± ◦ (ψi± )−1 : Bn → Rn is a smooth map with smooth inverse f : Rn → Bn given by
i

(x1 , . . . , xn )
f (x1 , . . . , xn ) = q .
2
|x| + 1

Hence, φi ◦ q|V ± ◦ (ψi± )−1 is a diffeomorphism, which means that q|V ± is a diffeomorphism.
i i

Problem 4.11. Show that a topological covering map is proper if and only if its fibres are finite, and therefore
the converse of Proposition 4.46 is false.
Let π : E → M be a topological covering map. Suppose that π is proper. Then {p} is compact for
every p ∈ M , which implies that π −1 (p) is compact in E. Let U be an evenly covered neighbourhood
of p. Then each component of π −1 (U ) contains exactly one point of π −1 (p) since each component maps
homeomorphically onto U . This collection of components then forms an open cover for π −1 (p), and must
therefore be finite by compactness. Thus, π −1 (p) is finite.
Conversely, suppose that π −1 (p) is finite for each p ∈ M . Let K ⊆ M be compact and let {Uα }α∈A be an
open covering of π −1 (K). Let p ∈ K be arbitrary, and let π −1 (p) = {q1p , . . . , qkpp }. By the fact that π is an
open map (topological covering maps are open) and π −1 (p) is finite, we can choose open neighbourhoods Vip

30
of qip such that Vip ⊆ Uα for each i = 1, . . . , kp and for some α ∈ A, and such that π|Vip is a homeomorphism
onto its image W p = π(Vip ). By compactness of K, let {W p1 , . . . , W pn } be an open covering of K. We
claim that collection of components of all the π −1 (W pi ) forms a finite open covering of π −1 (K). To see
this, let q ∈ π −1 (K), so that π(q) ∈ K. Then π(q) ∈ W pj for some j ∈ {1, . . . , n}. Then q is in one of
the components of π −1 (W pj ), as claimed. The fact that the covering is finite follows from the fact that the
p
fibres are finite. For each component Vi j of W pj , choose a Uα that contains it. This finite subcollection of
{Uα }α∈A is then a finite subcover of π (K). Hence, π −1 (K) is compact, which proves that π is proper.
−1

Since  : R → S1 defined in Example 2.13(b) is a smooth covering map with (countably) infinite fibres, it
is not proper. Hence, the converse of Proposition 4.46 is false.
Problem 4.12. Using the covering map 2 : R2 → T2 (see Example 4.35), show that the immersion
X : R2 → R3 defined in Example 4.2(d) descends to a smooth embedding of T2 into R3 . Specifically,
e : T2 → R3 , and then show that X
show that X passes to the quotient to define a smooth map X e is a smooth
embedding whose image is the given surface of revolution.
It is straightforward to check that X is constant on the fibres of 2 . Since 2 is a smooth submersion,
Theorem 4.30 guarantees that there is a smooth map X e making

R2
X
2

T2 R3
X
e

commute. Because 2 is actually a local diffeomorphism (Example 4.11) and X is a smooth immersion, it
follows that Xe must be a smooth immersion. More specifically, at each p ∈ R2 , we have dX e2 (p) ◦ d2p = dXp .
Since d2p is invertible and dXp is injective, it follows that dX e2 (p) is injective for every p; surjectivity
2
of  then implies that X e is an immersion. We will now show that X e is an injection. To this end, let
e 2πis1 , e2πis2 ) = X(e
X(e e 2πit1 , e2πit2 ). Then, by the definition of X,

(2 + cos 2πs1 ) cos 2πs2 = (2 + cos 2πt1 ) cos 2πt2


(2 + cos 2πs1 ) sin 2πs2 = (2 + cos 2πt1 ) sin 2πt2
sin 2πs1 = sin 2πt1 .

Since 2 + cos x is never zero, the first two equations imply that cos 2πs2 = cos 2πt2 and sin 2πs2 = sin 2πt2 .
Since cos x and sin x are never zero for the same x, at least one of the first two equations yields 2+cos 2πs1 =
2 + cos 2πt1 , which gives cos 2πs1 = cos 2πt1 . Putting this together with the third equation, we have that
(e2πis1 , e2πis2 ) = (e2πit1 , e2πit2 ), which proves injectivity.
Since T2 is compact and X e : T2 → R3
e is an injective smooth immersion, Proposition 4.22 implies that X
is an embedding.
Problem 4.13. Define a map F : S2 → R4 by F (x, y, z) = (x2 − y 2 , xy, xz, yz). Using the smooth covering
map of Example 2.13(f ) and Problem 4.10, show that F descends to a smooth embedding of RP2 into R4 .
Since F = G ◦ ι, where G : R3 → R4 is the map given by the same formula as F but extended to all of R3
and ι : S2 ,→ R3 is the inclusion, we have that F is smooth. Note that q : S2 → RP2 is a smooth submersion
and that F is constant on the fibres q −1 (x, y, x) = {±(x, y, z)}. By Theorem 4.30, there is then a map Fe
such that
S2
q F

RP2 R4
F
e

commutes. The commutativity of the diagram forces


1
Fe[x, y, z] = (x2 − y 2 , xy, xz, yz).
x2 + y 2 + z 2

31
In the coordinates of the usual chart (U1 , φ1 ), we have
1
Fe ◦ φ−1
1 (u, v) = F [1, u, v] =
e (1 − u2 , u, v, uv),
1 + u2 + v 2

and the matrix representation of dFep for p ∈ U1 is then

−4u − 2uv 2 1 − u2 + v 2 v − uv +3
 
1 −2uv
.
(1 + u2 + v 2 )2 2u2 v − 2v −2uv 1 + u2 − v 2 u + u3 − uv 2

The first minor is (up to a nonzero factor)



−4u − 2uv 2 1 − u2 + v 2
2 = 2v(1 + 2u2 + v 2 + u4 + u2 v 2 ),
2u v − 2v −2uv

which can only be zero if v = 0. But plugging v = 0 into the matrix representation of dFe shows that the
rows are linearly independent. Thus, Fe is an immersion on U1 . Similar computations show that Fe is an
immersion on the other charts.
We now show that Fe is injective. To this end, let Fe[x, y, x] = Fe[u, v, w]. We may assume that (x, y, z)
and (u, v, w) are normalized. Then

x2 − y 2 = u2 − v 2 , xy = uv, xz = uw, yz = vw.

Using these equations along with x2 + y 2 + z 2 = u2 + v 2 + w2 = 1, it can be shown that (x, y, z) = ±(u, v, w),
and therefore that [x, y, z] = [u, v, w]. Then Fb is an injective smooth submersion with compact domain, and
is therefore an embedding. Hence, Fe : RP2 → R4 is an embedding.

32

You might also like